Re: Is Many Worlds Falsifiable?

2023-09-06 Thread John Clark
On Wed, Sep 6, 2023 at 1:20 PM Jesse Mazer  wrote:

*> I tend to agree with Deutsch's intuitions on this but I think it gets
> into philosophical questions like whether the pilot wave being in some
> computational sense equivalent to MWI means that observers in other
> branches are "real", have their own distinct conscious experiences etc. It
> seems like it's at least a coherent philosophical interpretation of QM to
> postulate that only brain states corresponding to actual particle
> positions/movements in Bohmian mechanics give rise to conscious
> experiences, even though this seems very contrived and implausible to me.*
>

It does seem contrived because the situation is symmetrical, somebody in
one of those other universes could say the same thing about us.
  John K ClarkSee what's on my new list at  Extropolis

dss

-- 
You received this message because you are subscribed to the Google Groups 
"Everything List" group.
To unsubscribe from this group and stop receiving emails from it, send an email 
to everything-list+unsubscr...@googlegroups.com.
To view this discussion on the web visit 
https://groups.google.com/d/msgid/everything-list/CAJPayv2KbKX4hgMXMjoxPKKE-%2BGj2hn2nr%2Bei-fe0Q%3DNzM4jcQ%40mail.gmail.com.


Re: Is Many Worlds Falsifiable?

2023-09-06 Thread Jesse Mazer
I tend to agree with Deutsch's intuitions on this but I think it gets into
philosophical questions like whether the pilot wave being in some
computational sense equivalent to MWI means that observers in other
branches are "real", have their own distinct conscious experiences etc. It
seems like it's at least a coherent philosophical interpretation of QM to
postulate that only brain states corresponding to actual particle
positions/movements in Bohmian mechanics give rise to conscious
experiences, even though this seems very contrived and implausible to me.

On Wed, Sep 6, 2023 at 1:06 PM John Clark  wrote:

> On Wed, Sep 6, 2023 at 12:38 PM Jesse Mazer  wrote:
>
> *> Whether violations of Leggett-Garg inequalities rule out nonlocal
>> realistic theories seems to be a matter of definition, the inequality is
>> violated in Bohmian mechanics which is often referred to as a nonlocal
>> realistic theory,*
>>
>
>  David Deutsch, one of the leading advocates of Many Worlds, said  "*the
> de Broglie–Bohm theory, is a parallel universe theory in denial*", he
> thinks it's basically saying the same thing but just uses more euphemisms
> so as to be less upsetting to people. He says the pilot wave theory "*boils
> down to a probability field that acts like particles in other universes
> interfering with particles in our universe*".
>
>  John K ClarkSee what's on my new list at  Extropolis
> 
> dxd
>
>
>
>
> --
> You received this message because you are subscribed to the Google Groups
> "Everything List" group.
> To unsubscribe from this group and stop receiving emails from it, send an
> email to everything-list+unsubscr...@googlegroups.com.
> To view this discussion on the web visit
> https://groups.google.com/d/msgid/everything-list/CAJPayv0zzV0ua9uKe_H%2B3UNUDALMPwgEuKYWQZrhrV%2BN_y8TdA%40mail.gmail.com
> 
> .
>

-- 
You received this message because you are subscribed to the Google Groups 
"Everything List" group.
To unsubscribe from this group and stop receiving emails from it, send an email 
to everything-list+unsubscr...@googlegroups.com.
To view this discussion on the web visit 
https://groups.google.com/d/msgid/everything-list/CAPCWU3KHg%2B2T8EBmDsjdTm2v-Bjk0dGZv5d2kYgzR1UJw0%3Dfbw%40mail.gmail.com.


Re: Is Many Worlds Falsifiable?

2023-09-06 Thread John Clark
On Wed, Sep 6, 2023 at 12:38 PM Jesse Mazer  wrote:

*> Whether violations of Leggett-Garg inequalities rule out nonlocal
> realistic theories seems to be a matter of definition, the inequality is
> violated in Bohmian mechanics which is often referred to as a nonlocal
> realistic theory,*
>

 David Deutsch, one of the leading advocates of Many Worlds, said  "*the de
Broglie–Bohm theory, is a parallel universe theory in denial*", he thinks
it's basically saying the same thing but just uses more euphemisms so as to
be less upsetting to people. He says the pilot wave theory "*boils down to
a probability field that acts like particles in other universes interfering
with particles in our universe*".

 John K ClarkSee what's on my new list at  Extropolis

dxd

-- 
You received this message because you are subscribed to the Google Groups 
"Everything List" group.
To unsubscribe from this group and stop receiving emails from it, send an email 
to everything-list+unsubscr...@googlegroups.com.
To view this discussion on the web visit 
https://groups.google.com/d/msgid/everything-list/CAJPayv0zzV0ua9uKe_H%2B3UNUDALMPwgEuKYWQZrhrV%2BN_y8TdA%40mail.gmail.com.


Re: Is Many Worlds Falsifiable?

2023-09-06 Thread Jesse Mazer
Whether violations of Leggett-Garg inequalities rule out nonlocal realistic
theories seems to be a matter of definition, the inequality is violated in
Bohmian mechanics which is often referred to as a nonlocal realistic
theory, see the discussion on p. 12 of https://arxiv.org/pdf/1412.6139.pdf
-- apparently Leggett/Garg were saying violations of the inequalities rule
out nonlocal theories that lack macroscopic superpositions, but they
defined this in such a way that the Bohm interpretation *would* involve
macroscopic superpositions despite the particles having unique locations at
all times.

On Tue, Sep 5, 2023 at 10:59 PM John Clark  wrote:

>
> On Tue, Sep 5, 2023 at 10:34 PM Bruce Kellett 
> wrote:
>
> *>>> The Bell inequality can be derived without assuming realism*
>>>
>>>
>>> >> Everybody is wrong from time to time, but some people just can't
>>> admit it.
>>>
>>
>> *>I am sorry that you think John Bell was wrong..*
>>
>
> The violation of Bell's Inequality proves that things are not realistic
> or not local or both, but there is another inequality called  Leggett's
> inequality involving linear and elliptical polarized light that can
> narrow down that uncertainty. Leggett found his inequality in 2003 and it
> was experimentally proven to be violated in 2010. Nature is probably the
> best scientific journal in the world but I'm sure you'll say it's wrong
> just as you claim that Wikipedia was wrong because it says that you are
> incorrect and that the world is BOTH nonlocal AND non-realistic.
>
> "*Bell's inequality is established based on local realism. The violation
> of Bell's inequality by quantum mechanics implies either locality or
> realism or both are untenable. Leggett's inequality is derived based on
> nonlocal realism. The violation of Leggett's inequality implies that
> quantum mechanics is neither local realistic nor nonlocal realistic.*"
>
>
> Testing Leggett's Inequality Using Aharonov-Casher Effect
> 
>
>  By now I think you know you were wrong, but of course you will never
> admit it.
>
>   John K ClarkSee what's on my new list at  Extropolis
> 
> nvb
>
>
> --
> You received this message because you are subscribed to the Google Groups
> "Everything List" group.
> To unsubscribe from this group and stop receiving emails from it, send an
> email to everything-list+unsubscr...@googlegroups.com.
> To view this discussion on the web visit
> https://groups.google.com/d/msgid/everything-list/CAJPayv1hhWfRoKLTLSxjsYznGM3wJ_uMu90mH646F3p9Zj6urg%40mail.gmail.com
> 
> .
>

-- 
You received this message because you are subscribed to the Google Groups 
"Everything List" group.
To unsubscribe from this group and stop receiving emails from it, send an email 
to everything-list+unsubscr...@googlegroups.com.
To view this discussion on the web visit 
https://groups.google.com/d/msgid/everything-list/CAPCWU3%2By%3DiBF%2BSmru_f9ZntrivnUmv4TtKTN%3DRqJt%3D%3DzbGffCQ%40mail.gmail.com.


Re: Is Many Worlds Falsifiable?

2023-09-06 Thread John Clark
On Wed, Sep 6, 2023 at 7:40 AM Bruce Kellett  wrote:

 "*Bell's inequality is established based on local realism."*

>>>
>>> *>>> False.*
>>>
>>
>>  >> I didn't say that, the science journal Nature said that. So now
>> according to you not only is Wikipedia wrong but so is the science journal
>> Nature, the oldest and most prestigious science journal in the world. Do
>> you really think that people should believe you and not them?  Bruce,
>> nobody wins every argument, with this constant denial in the face of
>> mounting evidence you're starting to make a fool of yourself.
>>
> Testing Leggett's Inequality Using Aharonov-Casher Effect
>> 
>>
>
> *> You made a fool of yourself a long time ago. You didn't read Bell's
> papers with sufficient attention, if at all.*
>

It wasn't just me, according to you the people at the science journal
Nature and Wikipedia didn't read Bell's paper with sufficient attention
either, but of course you did. Do you really expect other people to believe
that? Do you even believe it yourself?
  John K ClarkSee what's on my new list at  Extropolis

wjm



> *> MWI is both non-realistic and non-local.*
>>>
>>
>> I agree, so what are we arguing about? Yes MWI is both non-realistic and
>> non-local, if it was not it would not have passed BOTH the Bell and the 
>> Leggett
>> inequality and it would no longer be in agreement with all known
>> experiments and it would no longer be a viable interpretation of Quantum
>> Mechanics.
>>
>
>

-- 
You received this message because you are subscribed to the Google Groups 
"Everything List" group.
To unsubscribe from this group and stop receiving emails from it, send an email 
to everything-list+unsubscr...@googlegroups.com.
To view this discussion on the web visit 
https://groups.google.com/d/msgid/everything-list/CAJPayv326pdgeA9eLx_TVC9sv9%3DOo6uTQ9mGQp4%2ByqQ8fHqQ1g%40mail.gmail.com.


Re: Is Many Worlds Falsifiable?

2023-09-06 Thread Bruce Kellett
On Wed, Sep 6, 2023 at 9:12 PM John Clark  wrote:

> On Wed, Sep 6, 2023 at 12:38 AM Bruce Kellett 
> wrote:
>
> The violation of Bell's Inequality proves that things are not realistic
>>> or not local or both,
>>>
>>
>> *> I have said that and you denied it.*
>>
>
> Show me where I denied that!!  I had been saying that things are not
> realistic or not local or both for well over a decade, but thanks to
> Leggett I now know that the answer is BOTH.
>
>
>> *> QM is non-realistic anyway.*
>>
>
> The experimental violation of Bell's Inequality proves that any theory
> that hopes to explain how the world works (QM for example) must be not
> realistic or not local or both, but the experimental violation of Leggett's
> inequality proves that any theory that hopes to explain how the world works
> must be *BOTH* nonlocal *AND* non-realistic, period. QM and MWI pass both
> the Bell and the Leggett test, that doesn't prove that either is correct
> but it does prove that whatever theory turns out to be true cannot be local
> and cannot be realistic. And neither test is able to prove that QM or the
> MWI is wrong.
>
>
>> >> "*Bell's inequality is established based on local realism."*
>>>
>>
>> *> False.*
>>
>
>  I didn't say that, the science journal Nature said that. So now according
> to you not only is Wikipedia wrong but so is the science journal Nature,
> the oldest and most prestigious science journal in the world. Do you
> really think that people should believe you and not them?  Bruce, nobody
> wins every argument, with this constant denial in the face of mounting
> evidence you're starting to make a fool of yourself.
>

You made a fool of yourself a long time ago. You didn't read Bell's papers
with sufficient attention, if at all.

Bruce



> Testing Leggett's Inequality Using Aharonov-Casher Effect
> 
>
>
>> *> MWI is both non-realistic and non-local.*
>>
>
> I agree, so what are we arguing about? Yes MWI is both non-realistic and
> non-local, if it was not it would not have passed BOTH the Bell and the 
> Leggett
> inequality and it would no longer be in agreement with all known
> experiments and it would no longer be a viable interpretation of Quantum M
> echanics.
>

-- 
You received this message because you are subscribed to the Google Groups 
"Everything List" group.
To unsubscribe from this group and stop receiving emails from it, send an email 
to everything-list+unsubscr...@googlegroups.com.
To view this discussion on the web visit 
https://groups.google.com/d/msgid/everything-list/CAFxXSLQjBJyKu%2BiQYcQxVafz1rPaHnewBQxrT%3DKBpDHTciZhJQ%40mail.gmail.com.


Re: Is Many Worlds Falsifiable?

2023-09-06 Thread John Clark
On Wed, Sep 6, 2023 at 12:38 AM Bruce Kellett  wrote:

The violation of Bell's Inequality proves that things are not realistic or
>> not local or both,
>>
>
> *> I have said that and you denied it.*
>

Show me where I denied that!!  I had been saying that things are not
realistic or not local or both for well over a decade, but thanks to
Leggett I now know that the answer is BOTH.


> *> QM is non-realistic anyway.*
>

The experimental violation of Bell's Inequality proves that any theory that
hopes to explain how the world works (QM for example) must be not realistic
or not local or both, but the experimental violation of Leggett's
inequality proves that any theory that hopes to explain how the world works
must be *BOTH* nonlocal *AND* non-realistic, period. QM and MWI pass both
the Bell and the Leggett test, that doesn't prove that either is correct
but it does prove that whatever theory turns out to be true cannot be local
and cannot be realistic. And neither test is able to prove that QM or the
MWI is wrong.


> >> "*Bell's inequality is established based on local realism."*
>>
>
> *> False.*
>

 I didn't say that, the science journal Nature said that. So now according
to you not only is Wikipedia wrong but so is the science journal Nature,
the oldest and most prestigious science journal in the world. Do you really
think that people should believe you and not them?  Bruce, nobody wins
every argument, with this constant denial in the face of mounting evidence
you're starting to make a fool of yourself.

Testing Leggett's Inequality Using Aharonov-Casher Effect



> *> MWI is both non-realistic and non-local.*
>

I agree, so what are we arguing about? Yes MWI is both non-realistic and
non-local, if it was not it would not have passed BOTH the Bell and the Leggett
inequality and it would no longer be in agreement with all known
experiments and it would no longer be a viable interpretation of Quantum M
echanics.

  John K ClarkSee what's on my new list at  Extropolis

nwe

-- 
You received this message because you are subscribed to the Google Groups 
"Everything List" group.
To unsubscribe from this group and stop receiving emails from it, send an email 
to everything-list+unsubscr...@googlegroups.com.
To view this discussion on the web visit 
https://groups.google.com/d/msgid/everything-list/CAJPayv3KAqu%3DgnOYVid-2zQJ07Xzu4TzekjCjicNay-oUYrDhQ%40mail.gmail.com.


Re: Is Many Worlds Falsifiable?

2023-09-05 Thread Bruce Kellett
On Wed, Sep 6, 2023 at 12:59 PM John Clark  wrote:

> On Tue, Sep 5, 2023 at 10:34 PM Bruce Kellett 
> wrote:
>
> *>>> The Bell inequality can be derived without assuming realism*
>>>
>>>
>>> >> Everybody is wrong from time to time, but some people just can't
>>> admit it.
>>>
>>
>> *>I am sorry that you think John Bell was wrong..*
>>
>
> The violation of Bell's Inequality proves that things are not realistic
> or not local or both,
>

I have said that and you denied it. QM is non-realistic anyway. The quantum
violation of the Bell inequalities show that it is also non-local. So it is
neither realistic nor local. No problems there. The violations of realism
have nothing to do with Bell, since his derivation does not assume realism,
despite your claims to the contrary.

but there is another inequality called  Leggett's inequality involving
> linear and elliptical polarized light that can narrow down that
> uncertainty. Leggett found his inequality in 2003 and it was
> experimentally proven to be violated in 2010. Nature is probably the best
> scientific journal in the world but I'm sure you'll say it's wrong just as
> you claim that Wikipedia was wrong because it says that you are incorrect
> and that the world is BOTH nonlocal AND non-realistic.
>
> "*Bell's inequality is established based on local realism.*
>

False.

*The violation of Bell's inequality by quantum mechanics implies either
> locality or realism or both are untenable. Leggett's inequality is derived
> based on nonlocal realism.*
>

Whatever that might be. But it seems to be based on a form of realism,
certainly.

* The violation of Leggett's inequality implies that quantum mechanics is
> neither local realistic nor nonlocal realistic.*"
>
>
> Testing Leggett's Inequality Using Aharonov-Casher Effect
> 
>
>  By now I think you know you were wrong, but of course you will never
> admit it.
>

If you want to prove me wrong, give a local account of the violation of the
Bell inequalities in non-realistic many worlds. MWI is both non-realistic
and non-local.

Bruce

-- 
You received this message because you are subscribed to the Google Groups 
"Everything List" group.
To unsubscribe from this group and stop receiving emails from it, send an email 
to everything-list+unsubscr...@googlegroups.com.
To view this discussion on the web visit 
https://groups.google.com/d/msgid/everything-list/CAFxXSLSzzBcO4S8G4h6trRPdSuPx%3D%3DHVcakFiNe3gkaPiCrGZw%40mail.gmail.com.


Re: Is Many Worlds Falsifiable?

2023-09-05 Thread John Clark
On Tue, Sep 5, 2023 at 10:34 PM Bruce Kellett  wrote:

*>>> The Bell inequality can be derived without assuming realism*
>>
>>
>> >> Everybody is wrong from time to time, but some people just can't
>> admit it.
>>
>
> *>I am sorry that you think John Bell was wrong..*
>

The violation of Bell's Inequality proves that things are not realistic or
not local or both, but there is another inequality called  Leggett's
inequality involving linear and elliptical polarized light that can narrow
down that uncertainty. Leggett found his inequality in 2003 and it was
experimentally proven to be violated in 2010. Nature is probably the best
scientific journal in the world but I'm sure you'll say it's wrong just as
you claim that Wikipedia was wrong because it says that you are incorrect
and that the world is BOTH nonlocal AND non-realistic.

"*Bell's inequality is established based on local realism. The violation of
Bell's inequality by quantum mechanics implies either locality or realism
or both are untenable. Leggett's inequality is derived based on nonlocal
realism. The violation of Leggett's inequality implies that quantum
mechanics is neither local realistic nor nonlocal realistic.*"


Testing Leggett's Inequality Using Aharonov-Casher Effect


 By now I think you know you were wrong, but of course you will never admit
it.

  John K ClarkSee what's on my new list at  Extropolis

nvb

-- 
You received this message because you are subscribed to the Google Groups 
"Everything List" group.
To unsubscribe from this group and stop receiving emails from it, send an email 
to everything-list+unsubscr...@googlegroups.com.
To view this discussion on the web visit 
https://groups.google.com/d/msgid/everything-list/CAJPayv1hhWfRoKLTLSxjsYznGM3wJ_uMu90mH646F3p9Zj6urg%40mail.gmail.com.


Re: Is Many Worlds Falsifiable?

2023-09-05 Thread Bruce Kellett
On Wed, Sep 6, 2023 at 12:14 PM John Clark  wrote:

> On Tue, Sep 5, 2023 at 7:40 PM Bruce Kellett 
> wrote:
>
> *> The Bell inequality can be derived without assuming realism*
>
>
> Everybody is wrong from time to time, but some people just can't admit it.
>
>

I am sorry that you think John Bell was wrong..

Bruce

-- 
You received this message because you are subscribed to the Google Groups 
"Everything List" group.
To unsubscribe from this group and stop receiving emails from it, send an email 
to everything-list+unsubscr...@googlegroups.com.
To view this discussion on the web visit 
https://groups.google.com/d/msgid/everything-list/CAFxXSLRQVRJJ77TTKXEADbVL-qS5ZuCuGELGEKcd6meDPi5Wvg%40mail.gmail.com.


Re: Is Many Worlds Falsifiable?

2023-09-05 Thread John Clark
On Tue, Sep 5, 2023 at 7:40 PM Bruce Kellett  wrote:

*> The Bell inequality can be derived without assuming realism*


Everybody is wrong from time to time, but some people just can't admit it.

  John K ClarkSee what's on my new list at  Extropolis

urw




>> >> Huh? How can you "*have **read quite extensively on Bell's theorem
 and locality*" and not know that Bell's theorem is a test to see if
 any theory that assumes* local realism* can account for experimental
 observations? Hell if you did nothing but skim the Wikipedia article on 
 Bell's
 theorem you should know that because the very first sentence is:
 *"Bell's theorem is a term encompassing a number of closely related
 results in physics, all of which determine that quantum mechanics is
 incompatible with local hidden-variable theories"*
 And just a few sentences later Wikipedia says:
 *"Its derivation here depends upon two assumptions: first, that the
 underlying physical properties and exist independently of being observed or
 measured (sometimes called the assumption of realism); and second, that
 Alice's choice of action cannot influence Bob's result or vice versa (often
 called the assumption of locality)"*

>>>
>>>

-- 
You received this message because you are subscribed to the Google Groups 
"Everything List" group.
To unsubscribe from this group and stop receiving emails from it, send an email 
to everything-list+unsubscr...@googlegroups.com.
To view this discussion on the web visit 
https://groups.google.com/d/msgid/everything-list/CAJPayv08F%2B%2BLENzECL7G5AKp0OQK3BMw-0Eyeh4%3DEOoj25VLcQ%40mail.gmail.com.


Re: Is Many Worlds Falsifiable?

2023-09-05 Thread Bruce Kellett
On Wed, Sep 6, 2023 at 9:31 AM John Clark  wrote:

> On Tue, Sep 5, 2023 at 7:06 PM Bruce Kellett 
> wrote:
>
> >> Huh? How can you "*have **read quite extensively on Bell's theorem and
>>> locality*" and not know that Bell's theorem is a test to see if any
>>> theory that assumes* local realism* can account for experimental
>>> observations? Hell if you did nothing but skim the Wikipedia article on 
>>> Bell's
>>> theorem you should know that because the very first sentence is:
>>> *"Bell's theorem is a term encompassing a number of closely related
>>> results in physics, all of which determine that quantum mechanics is
>>> incompatible with local hidden-variable theories"*
>>> And just a few sentences later Wikipedia says:
>>> *"Its derivation here depends upon two assumptions: first, that the
>>> underlying physical properties and exist independently of being observed or
>>> measured (sometimes called the assumption of realism); and second, that
>>> Alice's choice of action cannot influence Bob's result or vice versa (often
>>> called the assumption of locality)"*
>>>
>>
>> > *Unfortunately, Wikipedia is not an authoritative source.* [...]   *as
>> I have said several times, "realism" has nothing to do with it.*
>>
>
> So let's see, Wikipedia is wrong, John Stewart Bell is wrong, and
> high school algebra is wrong, but Bruce Kellett is absolutely positively
> 100% correct. Have I got that about right?
>

Get a grip, John. That is not what I said. The Bell inequality can be
derived without assuming realism, so realism is irrelevant to the issue.


*> In fact, the assumption of realism is pretty meaningless because QM
>> itself does not have this property -- it is intrinsically probabilistic and
>> non-realist.*
>
>
> What are you talking about? The non-existence of a property does not
> render it meaningless, dragons don't exist but I know what the word means,
> it's not gibberish. And like Quantum Mechanics Many Worlds is also
> non-realistic, good thing too because otherwise it wouldn't match
> experimental results.
>

You really have lost the plot, haven't you!

Bruce

-- 
You received this message because you are subscribed to the Google Groups 
"Everything List" group.
To unsubscribe from this group and stop receiving emails from it, send an email 
to everything-list+unsubscr...@googlegroups.com.
To view this discussion on the web visit 
https://groups.google.com/d/msgid/everything-list/CAFxXSLT9Vd%3D_fi-xLbTN3GOka98GH9TqQcBUOsuc75d2GxZFJw%40mail.gmail.com.


Re: Is Many Worlds Falsifiable?

2023-09-05 Thread John Clark
On Tue, Sep 5, 2023 at 7:06 PM Bruce Kellett  wrote:



>> Huh? How can you "*have **read quite extensively on Bell's theorem and
>> locality*" and not know that Bell's theorem is a test to see if any
>> theory that assumes* local realism* can account for experimental
>> observations? Hell if you did nothing but skim the Wikipedia article on 
>> Bell's
>> theorem you should know that because the very first sentence is:
>> *"Bell's theorem is a term encompassing a number of closely related
>> results in physics, all of which determine that quantum mechanics is
>> incompatible with local hidden-variable theories"*
>> And just a few sentences later Wikipedia says:
>> *"Its derivation here depends upon two assumptions: first, that the
>> underlying physical properties and exist independently of being observed or
>> measured (sometimes called the assumption of realism); and second, that
>> Alice's choice of action cannot influence Bob's result or vice versa (often
>> called the assumption of locality)"*
>>
>
> > *Unfortunately, Wikipedia is not an authoritative source.* [...]   *as
> I have said several times, "realism" has nothing to do with it.*
>

So let's see, Wikipedia is wrong, John Stewart Bell is wrong, and high
school algebra is wrong, but Bruce Kellett is absolutely positively 100%
correct. Have I got that about right?

*> In fact, the assumption of realism is pretty meaningless because QM
> itself does not have this property -- it is intrinsically probabilistic and
> non-realist.*


What are you talking about? The non-existence of a property does not render
it meaningless, dragons don't exist but I know what the word means, it's
not gibberish. And like Quantum Mechanics Many Worlds is also
non-realistic, good thing too because otherwise it wouldn't match
experimental results.

  John K ClarkSee what's on my new list at  Extropolis

nwm


>

-- 
You received this message because you are subscribed to the Google Groups 
"Everything List" group.
To unsubscribe from this group and stop receiving emails from it, send an email 
to everything-list+unsubscr...@googlegroups.com.
To view this discussion on the web visit 
https://groups.google.com/d/msgid/everything-list/CAJPayv0ALk7pdr2Z0KmVC3VYMA_UBOZwE9-bO7L4%2Bo9d6FY3gg%40mail.gmail.com.


Re: Is Many Worlds Falsifiable?

2023-09-05 Thread Bruce Kellett
On Tue, Sep 5, 2023 at 8:34 PM John Clark  wrote:

> On Mon, Sep 4, 2023 at 8:14 PM Bruce Kellett 
> wrote:
>
>> On Tue, Sep 5, 2023 at 12:02 AM smitra  wrote:
>>
>
>
> >> Bell's theorem is about local hidden variables theories
>>
>>
>> > *It is difficult to know how to respond to this absurd idea. I have
>> read quite extensively on Bell's theorem and locality in quantum mechanics
>> and I have never met this contention before.*
>>
>
> Huh? How can you "*have **read quite extensively on Bell's theorem and
> locality*" and not know that Bell's theorem is a test to see if any
> theory that assumes* local realism* can account for experimental
> observations? Hell if you did nothing but skim the Wikipedia article on Bell's
> theorem you should know that because the very first sentence is:
>
> *"Bell's theorem is a term encompassing a number of closely related
> results in physics, all of which determine that quantum mechanics is
> incompatible with local hidden-variable theories"*
>
> And just a few sentences later Wikipedia says:
>
> *"Its derivation here depends upon two assumptions: first, that the
> underlying physical properties and exist independently of being observed or
> measured (sometimes called the assumption of realism); and second, that
> Alice's choice of action cannot influence Bob's result or vice versa (often
> called the assumption of locality)"*
>

Unfortunately, Wikipedia is not an authoritative source. The derivation of
the Bell inequality that you refer to in Wikipedia is not the derivation
given by Bell in his original papers. Bell's own derivation appears later
in the article, and you can see that Bell does not make the realism
assumption. Since the inequality can be derived without this assumption,
violating relaasm makes no difference to the overall result. The
correlations in any local theory must satisfy the inequality. Bell shows
that the quantum mechanical correlations violate the inequality, so quantum
mechanics cannot be a local theory, and any hidden variable completion of
QM must also be non-local. Other people have claimed that Bell made a whole
range of other assumptions that their pet theories violate, thus rendering
Be;ll's theorem toothless. But one is hard-pressed to see where any of
these supposed additional assumptions come in. In fact, the range of things
sometimes said to be assumed are often contradictory.

The important point is that Bell used a particular implementation of the
idea of locality for his theorem, and few other assumptions (the main one
being the absence of superdeterminism), leaving the consequence of
violations of the inequality pretty clear -- any such theory must be
non-local. Quantum mechanics violates the inequality, therefore quantum
mechanics is intrinsically non-local. Experiment confirms the quantum
mechanical predictions. But since the inequality itself does not depend on
any assumption of realism, the observed violations cannot be explained by
claiming that the theory is local but non-realistic -- as I have said
several times, "realism" has nothing to do with it. The Wikipedia article
is quite misleading in this respect because it does not make clear that the
result can also be derived without assuming realism (measurement results
exist in the state *before* the measurement is performed.) In fact, the
assumption of realism is pretty meaningless because QM itself does not have
this property -- it is intrinsically probabilistic and non-realist.

Bruce

-- 
You received this message because you are subscribed to the Google Groups 
"Everything List" group.
To unsubscribe from this group and stop receiving emails from it, send an email 
to everything-list+unsubscr...@googlegroups.com.
To view this discussion on the web visit 
https://groups.google.com/d/msgid/everything-list/CAFxXSLTUV-9kzw6tWaGh0t2DdQ45Uzr2guq435kR7wrSyyxJjA%40mail.gmail.com.


Re: Is Many Worlds Falsifiable?

2023-09-05 Thread John Clark
On Mon, Sep 4, 2023 at 8:14 PM Bruce Kellett  wrote:

> On Tue, Sep 5, 2023 at 12:02 AM smitra  wrote:
>


>> Bell's theorem is about local hidden variables theories
>
>
> > *It is difficult to know how to respond to this absurd idea. I have
> read quite extensively on Bell's theorem and locality in quantum mechanics
> and I have never met this contention before.*
>

Huh? How can you "*have **read quite extensively on Bell's theorem and
locality*" and not know that Bell's theorem is a test to see if any theory
that assumes* local realism* can account for experimental observations?
Hell if you did nothing but skim the Wikipedia article on Bell's theorem
you should know that because the very first sentence is:

*"Bell's theorem is a term encompassing a number of closely related results
in physics, all of which determine that quantum mechanics is incompatible
with local hidden-variable theories"*

And just a few sentences later Wikipedia says:

*"Its derivation here depends upon two assumptions: first, that the
underlying physical properties and exist independently of being observed or
measured (sometimes called the assumption of realism); and second, that
Alice's choice of action cannot influence Bob's result or vice versa (often
called the assumption of locality)"*

And I might add that in the duel between theories that assume local realism
and quantum mechanics, experimental observation has determined that the
undisputed winner was quantum mechanics.

> standard QM has no explanation for the correlations


Yes. That has been the standard complaint about Quantum Mechanics since the
day it was invented, it can tell you what will happen with very high
precision but it can't tell you why, that's why quantum interpretation has
become a major industry and why very few ever felt there was a need for a
Newtonian interpretation. The leading interpretation, if you could even
call it an interpretation, is the one from Copenhagen which is so vague
it's not even wrong, the second most popular is "Shut Up And Calculate"
which works fine if you're only interested in engineering considerations,
the third most popular is Many Worlds which starts from the experimentally
derived *FACT* that things cannot be both local and realistic and then just
follows to where Schrodinger's Equation leads.  And it turns out it leads
to many worlds.


>
>>
>> * >> You seem to pretend that it's a theorem of QM, in which case one
>> would start from the postulates of QM and derive bounds on correlations for
>> any system described by a local Hamiltonian. That's obviously not true.*
>>
>
> > Strange, then, that John Bell managed to do that.
>

As I mentioned before, in John Bell's paper where he derived his inequality
he first assumed that things were both realistic and local and then just
used high school algebra and logic, he didn't use any Quantum Mechanics at
all to derive it, although he did show that his inequality was incompatible
with Quantum Mechanics. At the time he didn't know if his inequality was
true or not because it would be about two decades before it was
experimentally shown to be untrue. Since nobody believed that Bell's
algebra or logic was wrong the only conclusion was that the starting
assumption must be incorrect and things could not be locally realistic. If
it had been experimentally found that the inequality was true then that
would have proven that Quantum Mechanics made a wrong prediction and so
must be incomplete, but that's not the way things turned out.

  John K ClarkSee what's on my new list at  Extropolis

okx


>

-- 
You received this message because you are subscribed to the Google Groups 
"Everything List" group.
To unsubscribe from this group and stop receiving emails from it, send an email 
to everything-list+unsubscr...@googlegroups.com.
To view this discussion on the web visit 
https://groups.google.com/d/msgid/everything-list/CAJPayv0oVTMGybnMP0Fnj7pYnEx98QMzgZbyQtLHZ%3D2ScwBDww%40mail.gmail.com.


Re: Is Many Worlds Falsifiable?

2023-09-05 Thread 'scerir' via Everything List
According to John Bell, if A is one of the two wings of a typical Bell 
apparatus, i the observable to be measured in A and x its possible value, and 
if B is the other of the two wings, j is the observable to be measured in B and 
y its possible value, and if Lambda is the hidden-variable joint state 
description of the composite (entangled) quantum system, we can write the 
following
 
Bell factorisability condition
 
p_A,B,Lambda (x,y|i,j) = p_A,Lambda (x|i) p_B,Lambda (y|j)
 
which just means that the joint probability of outcomes x and y, for 
measurements of observables i and j, in the A and B wings, is equal to the 
product of the the separate probabilities.
 
We know that so many experiments have shown the expression above is far from 
reality. In other words it is well known that this factorisability condition is 
violated by quantum mechanics (QM).
 
Following Jarrett (and also Shimony, Howard, Cushing, Suppes, van Fraassen, and 
others) the Bell factorisability condition is equivalent to two independent 
conditions,
 
Locality Condition
 
p_A,Lambda (x|i,j) = p_A,Lambda (x|i)
 
p_B,Lambda (y|i,j) = p_B,Lambda (y|j)
 
Separability Condition
 
p_A,Lambda (x|i,j,y) = p_A,Lambda (x|i,j)
 
p_B,Lambda (y|i,j,x) = P_B,Lambda (y|i,j)
 
where Locality is defined as: Given two systems A and B, space-like separated, 
the state of A cannot be influenced by events (measurements) on B, and 
viceversa.
 
where Separability is defined as: Two systems, separated by some 
spatio-temporal interval, possess their own separate states, regardless of 
their previous history, and the joint state is completely determined by their 
own separate states.
 
Eberhard, Page, Shimony, Ghirardi (et al.) have shown that QM only implies the 
violation of the Separability condition (the world is non-separable, there is 
wholeness, there is some tao in physics) .
 
In other words it is possible to show (following Jarrett, Shimony, Ghirardi, 
Howard, Cushing, Eberhard, maybe van Fraassen, maybe Fine, etc.) that QM 
violates the Separability condition but does not violate the Locality condition.
 
In physical terms the above means that QM does not allow faster than light 
(FTL) signaling (Eberhard, Nuovo Cimento, 46B, 1978, 392; Ghirardi et al., 
Found. Phys., 23, 1993, 341).
 
It is possible to show (following Jarrett, Shimony, Ghirardi, Howard, Eberhard, 
Cushing, maybe van Fraassen, maybe Fine, etc.) that a “deterministic” theory 
(i.e. one in which the range of any probability distribution of outcomes is the 
set: 0 or 1) reproducing all the predictions of QM, does not violate the 
Separability Condition, but must violate the Locality Condition.
 
In fact the Separability Condition means that ...
 
p_A,Lambda (x|i,j,y) = p_A,Lambda (x|i,j)
 
p_B,Lambda (y|i,j,x) = P_B,Lambda (y|i,j)
 
so, if the specification of Lambda, i, j, in principle determines completely 
the outcomes x, y, then any additional conditioning on x or y is superfluous, 
having x and y just one value allowed, so they cannot affect the probability, 
which (in a deterministic theory) can take just the values 0 or 1.
 
Thus a *deterministic* QM can not violate the Separability Condition and must 
violate the Locality Condition, which means ... faster than light (FTL) 
signaling.
 

-- 
You received this message because you are subscribed to the Google Groups 
"Everything List" group.
To unsubscribe from this group and stop receiving emails from it, send an email 
to everything-list+unsubscr...@googlegroups.com.
To view this discussion on the web visit 
https://groups.google.com/d/msgid/everything-list/90209207.4416216.1693905304189%40mail1.libero.it.


Re: Is Many Worlds Falsifiable?

2023-09-04 Thread Bruce Kellett
On Tue, Sep 5, 2023 at 12:02 AM smitra  wrote:

> On 04-09-2023 01:35, Bruce Kellett wrote:
> > On Sun, Sep 3, 2023 at 11:37 AM smitra  wrote:
> >
> >> The time evolution according to
> >> the Schrödinger equation is manifestly local,
> >
> > But unitary evolution according to the SE cannot account for the
> > correlation of entangled particles.
> >
>
> It can, just calculate it and don't collapse the wavefunction.


OK. So show me this calculation that gives a local explanation of the
correlations.


>> Another important thing to note here is that Bell's theorem only
> >> applies to hidden variable theories, it does not apply to QM in general.
> >
> > Where on earth did you get that idea from? As John has pointed out,
> > Bell's theorem does not require even quantum mechanics. It is just a
> > piece of mathematics.It applies with complete generality to quantum
> > mechanics, with or without hidden variables.
> >
>
> Bell's theorem is about local hidden variables theories


It is difficult to know how to respond to this absurd idea. I have read
quite extensively on Bell's theorem and locality in quantum mechanics and I
have never met this contention before. Maybe 'scerir' has some reference to
it, but I have never seen such a suggestion. The point, it seems to me, is
that Bell's theorem concludes that any hidden variable completion of
quantum mechanics must be non-local. Since standard QM has no explanation
for the correlations, it might be supposed that some hidden variable
completion of the theory would work. However, Bell shows that even such a
hidden variable completion of the theory must be non-local. But this is the
case for any formulation of quantum mechanics -- one does not have to
assume the existence of hidden variables in order to derive the Bell
inequalities. The standard formulation of quantum mechanics explains the
correlations non-locally.

There is a simple argument for non-locality:
A) All local systems are separable (factorizable).
Hence all non-separable (non-factorizable) systems are non-local.
The entangled singlet state is non-separable. Therefore it is non-local.


What conclusions can we draw? If we assume that QM is not fundamental
> and that there exists a hidden variables theory that reproduces QM
> either exactly or to a good approximation, then we can conclude that
> such a hidden variables theory cannot be local.
>
> Or we can conclude that QM is fundamental and that there is no deeper
> hidden variables theory underlying QM. In this case the violation of
> Bell's inequality does not imply non-locality. However, collapse is then
> still a non-local mechanism.
>

If QM is fundamental and complete, then it must contain a local explanation
of the Bell correlations. No-one has ever been able to produce such an
explanation. Reality is, therefore, fundamentally non-local.


> Again, As I pointed out to John, even if you assume that Bell's
> > theorem does not apply to MWI (and of course it does), then it does
> > not follow that the theory is local. It could be non-local for reasons
> > unconnected with Bell's theorem.
>
>
> Yes, but the only source of non-locality is collapse. Once you get rid
> of collapse, the theory becomes local, because the Standard Model is a
> local theory.
>

And the standard Model (with or without collapse) cannot explain the
Bell-type correlations.



You seem to pretend that it's a theorem of QM, in which case one would
> start from the postulates of QM and derive bounds on correlations for
> any system described by a local Hamiltonian. That's obviously not true.
>

Strange, then, that John Bell managed to do that.

Bruce

-- 
You received this message because you are subscribed to the Google Groups 
"Everything List" group.
To unsubscribe from this group and stop receiving emails from it, send an email 
to everything-list+unsubscr...@googlegroups.com.
To view this discussion on the web visit 
https://groups.google.com/d/msgid/everything-list/CAFxXSLRAH%2Bx8bUk_NihBihdBbgBdbpbFC_hU_oiCfKQJ0T3boQ%40mail.gmail.com.


Re: Is Many Worlds Falsifiable?

2023-09-04 Thread smitra

On 04-09-2023 01:35, Bruce Kellett wrote:

On Sun, Sep 3, 2023 at 11:37 AM smitra  wrote:


On 31-08-2023 06:08, Bruce Kellett wrote:


That is all very well, but it is not a local account of violations

of

the Bell inequalities. You merely claim that the local theory is

such

an account, but you do not spell it out.


John has addressed this in a subsequent reply where he cites an old
reply giving the detailed account involving polarizers.


I have responded to John in a separate post. He appears to have a very
weak grasp of logic, and his arguments are not valid.


Thing is that in conventional QM we only have the dynamics only
involves
the Schrödinger equation and collapse.


The Schrodinger equation is not necessary for quantum mechanics. The
Heiseberg matrix formulation does not involve the SE. Time evolution
is just a unitary transformation after all. The wave function is not
necessary. Dirac, in his book on quantum mechanics, mentions the wave
function only in an inconsequential footnote.



It's equivalent, so it doesn;t matter that there exists an alternative 
formalism.



The time evolution according to
the Schrödinger equation is manifestly local,


But unitary evolution according to the SE cannot account for the
correlation of entangled particles.



It can, just calculate it and don't collapse the wavefunction. If the 
parallel worlds are unobservable FAPP, then it shouldn't matter whether 
or not you assume they exist or not, at least for these sorts of 
practical experiments.




while the collapse is the
only non-local part. So, any version of QM in which there is no
collapse
is guaranteed to be local.



Another important thing to note here is that Bell's theorem only
applies
to hidden variable theories, it does not apply to QM in general.


Where on earth did you get that idea from? As John has pointed out,
Bell's theorem does not require even quantum mechanics. It is just a
piece of mathematics.It applies with complete generality to quantum
mechanics, with or without hidden variables.



Bell's theorem is about local hidden variables theories, it's not a 
theorem of QM in the sense of something that follows from the postulates 
of QM like e.g Ehrenfest's theorem. It's theorem that follows from he 
assumed properties of a general local hidden variables theory and it 
derives bounds on correlations. You can then consider the correlations 
of certain observables in QM and see that they violate these 
inequalities.


What conclusions can we draw? If we assume that QM is not fundamental 
and that there exists a hidden variables theory that reproduces QM 
either exactly or to a good approximation, then we can conclude that 
such a hidden variables theory cannot be local.


Or we can conclude that QM is fundamental and that there is no deeper 
hidden variables theory underlying QM. In this case the violation of 
Bell's inequality does not imply non-locality. However, collapse is then 
still a non-local mechanism.






The MWI
is not a hidden variables theory, so Bell's theorem has nothing
whatsoever to say about this.


Again, As I pointed out to John, even if you assume that Bell's
theorem does not apply to MWI (and of course it does), then it does
not follow that the theory is local. It could be non-local for reasons
unconnected with Bell's theorem.



Yes, but the only source of non-locality is collapse. Once you get rid 
of collapse, the theory becomes local, because the Standard Model is a 
local theory.




We have had this discussion before, and you couldn't give the

detailed

local account then either.


You disputed the well established fact that all known interactions
are
locaThat is not a well establised fact. Given the violations of the
Bell inequalitiers, the only well established fact is that standard
QM is non-local.




As pointed out above the violation of Bell's inequality only implies 
non-locality in hidden variables models. Bell's theorem s a theorem 
derived from the general properties of an arbitrary local hidden 
variables theory and one then derives bounds for correlations.


You seem to pretend that it's a theorem of QM, in which case one would 
start from the postulates of QM and derive bounds on correlations for 
any system described by a local Hamiltonian. That's obviously not true.




You would not take a formal answer like

psi(x, t) = Exp(-i H/hbar t) psi(x,0)

where H is the a local Hamiltonian that describes the dynamics for
an
answer.


Of course that is not an answer. It is merely a re-stating of your
contention that QM is always local. Whether or not that Hamiltonian
formulation is able to account for the Bell-type correlations is
precisely the point at issue. Restating that the correlations do
indeed have a local explanation does not take us any further forward.



No non-local interactions have ever been demonstrated to exist.



You wanted me two explicitly write out H for a Bell-type
experiment for H a manifestly local Hamiltonian, and then to com

Re: Is Many Worlds Falsifiable?

2023-09-04 Thread John Clark
On Mon, Sep 4, 2023 at 7:29 AM Bruce Kellett  wrote:

* >>> Consider the following. shine a laser at the moon, then scan across
>>> the surface of the moon. The spot of light on the moon's surface clearly
>>> can move at any speed, particularly FTL. Now, if you use the laser to
>>> transmit a message to the first point, then scan away and re-transmit to
>>> the second location, you can certainly transmit information FTL.*
>>>
>>
>> *>> Don't be ridiculous! Light takes about 1 1/4 seconds to reach the
>> Moon, if I  aim a laser at point X on the Moon and then move it to point Y
>> also on the Moon it will take the usual 1 1/4 seconds after I moved my
>> laser before anybody at point X observes that the light coming from Earth
>> has gone off, and it will take the usual 1 1/4 seconds before anybody at
>> point Y sees a light from Earth go on, and 2 1/2 seconds before anybody on
>> planet Earth sees the spot of light at point X start to move. Nobody on the
>> Earth or on the Moon has received or transmitted any information faster
>> than light. If it was possible to transmit information FTL according to
>> relativity you could send a message into the past, you could talk to  the
>> Bruce Kellett of yesterday and that would create paradoxes.*
>>
>
> *> No. The example was not particularly well thought out.*
>

*That's true, your example wasn't particularly well thought out.*


> * > My point is that geometrical motions can exceed light velocity,*
>

*So what? That has nothing to do with the speed of causality or the maximum
speed that matter, energy or information can travel.  *

*> and distant galaxies recede at greater than light speed.*
>

*In General Relativity space is allowed to expand at any speed, but nothing
in space can move faster than light through that space, that's why we can't
see those distant galaxies and never will be able to. And that's why they
can no longer affect us in any way and we can no longer effect them. By the
way, why do you believe that those distant galaxies REALLY exist when you
don't believe that any of Everett's Many Worlds do? *



> *>> If Many Worlds is correct then if "you" (personal pronouns can become
>> problematic when talking about the multiverse) perform the polarizer
>> experiment on 1 million entangled photons then in the multiverse there are
>> 1 million new Bruce Kelletts that are absolutely identical in every way
>> EXCEPT for the fact that they each have 1 million different memories of how
>> those 1 million entangle protons behaved when they hit their polarizers.*
>>
>
> *> But for any one observer, even in many worlds, there is only ever one
> outcome for each experiment.*
>

*Yes but each experimenter has a different memory of how previous
experiments turned up, most were only slightly different but some were
radically different.   *

*> And the existence of other words does not affect the result that that
> individual observer obtains. Hence Bell's theorem applies separately for
> every individual, even in many worlds.*
>

*And I said precisely that in my previous email  "in all of them all the
Bruce Kelletts can experimentally confirm that Bell's Inequality can be
violated which would be logically impossible if things were both realistic
and local".*


> *>>> Bell's theorem applies equally to all the copies individually.*
>>>
>>
>> *>>Yes, and in all of them all the Bruce Kelletts can experimentally
>> confirm that Bell's Inequality can be violated which would be logically
>> impossible if things were both realistic and local. *
>>
>
> *>That dichotomy does not apply.*
>

*If  you really believe that then you are in effect claiming that John
Stewart Bell was wrong, and every mathematician and physicist on planet
Earth is wrong, and high school algebra is wrong,  Do you REALLY believe
that? Are you really that desperate to get rid of Many Worlds? *


*>> Entangled photons have opposite polarizations so if an entangled photon
>> of undetermined polarization hits a polarizer oriented in the up" direction
>> (what you call "up" could be any direction) and Many Worlds is correct then
>> the universe splits many times but in NO universe is there a case where 2
>> entangle photons both make it through polarizers oriented in the same
>> direction.*
>>
>
> *> That is one of the things that have to be explained.*
>

*Entangled photons are most commonly created by sending a single photon
through a crystal that has nonlinear optical characteristics, when that
happens the photon is destroyed and 2 entangled photons are produced, each
with exactly half the energy of the deceased parent photon. For the
conservation of angular momentum to be preserved the 2 photons MUST have
opposite polarizations. Probably the most common crystal to do this is Beta
Barium Borate (BaB2O4) but there are others.*


>
> *>> DO YOUR HOMEWORK! It's been known for hundreds of years that light
>> beams with opposite polarizations treat polarizers in opposite ways, and
>> it's been known since 1

Re: Is Many Worlds Falsifiable?

2023-09-04 Thread Jason Resch
As Rob Garrett shows here, there's really nothing mysterious about
entanglement.

Entanglement is merely measurement. The mystery, if there is one, is why
are measurements consistent across time:

https://youtu.be/dEaecUuEqfc?si=psmNck41LbAW4SjV

Jason

On Mon, Sep 4, 2023, 7:48 AM 'scerir' via Everything List <
everything-list@googlegroups.com> wrote:

>
>
> Il 04/09/2023 12:29 +01 Bruce Kellett  ha scritto:
>
> No. The example was not particularly well thought out. My point is that
> geometrical motions can exceed light velocity, and distant galaxies recede
> at greater than light speed. Light speed limits only physical transmission,
> unless by tachyons. In fine, *understanding non-locality probably
> involves refining our understanding of space and time.*
>
> https://www.edge.org/response-detail/26790
> Anton Zeilinger. “It appears that an understanding is possible via the
> notion of information. Information seen as the possibility of obtaining
> knowledge. Then quantum entanglement describes a situation where
> information exists about possible correlations between possible future
> results of possible future measurements without any information existing
> for the individual measurements. The latter explains quantum randomness,
> the first quantum entanglement. And both have significant consequences for
> our customary notions of causality. It remains to be seen what the
> consequences are for our notions of space and time, or space-time for that
> matter. *Space-time itself cannot be above or beyond such considerations.
> I suggest we need a new deep analysis of space-time, a conceptual analysis
> maybe analogous to the one done by the Viennese physicist-philosopher Ernst
> Mach who kicked Newton’s absolute space and absolute time form their
> throne.* The hope is that in the end we will have new physics analogous
> to Einstein’s new physics in the two theories of relativity.”
>
>
> --
> You received this message because you are subscribed to the Google Groups
> "Everything List" group.
> To unsubscribe from this group and stop receiving emails from it, send an
> email to everything-list+unsubscr...@googlegroups.com.
> To view this discussion on the web visit
> https://groups.google.com/d/msgid/everything-list/243069131.8543635.1693828098555%40mail1.libero.it
> 
> .
>

-- 
You received this message because you are subscribed to the Google Groups 
"Everything List" group.
To unsubscribe from this group and stop receiving emails from it, send an email 
to everything-list+unsubscr...@googlegroups.com.
To view this discussion on the web visit 
https://groups.google.com/d/msgid/everything-list/CA%2BBCJUjzCQ6prDvQ%3DwV7PjZaZCWt5ufjesyHRCAXibb-CDER-g%40mail.gmail.com.


Re: Is Many Worlds Falsifiable?

2023-09-04 Thread 'scerir' via Everything List
 

> Il 04/09/2023 12:29 +01 Bruce Kellett  ha scritto:
>  
> No. The example was not particularly well thought out. My point is that 
> geometrical motions can exceed light velocity, and distant galaxies recede at 
> greater than light speed. Light speed limits only physical transmission, 
> unless by tachyons. In fine, understanding non-locality probably involves 
> refining our understanding of space and time.
> 

https://www.edge.org/response-detail/26790

Anton Zeilinger. “It appears that an understanding is possible via the notion 
of information. Information seen as the possibility of obtaining knowledge. 
Then quantum entanglement describes a situation where information exists about 
possible correlations between possible future results of possible future 
measurements without any information existing for the individual measurements. 
The latter explains quantum randomness, the first quantum entanglement. And 
both have significant consequences for our customary notions of causality. It 
remains to be seen what the consequences are for our notions of space and time, 
or space-time for that matter. Space-time itself cannot be above or beyond such 
considerations. I suggest we need a new deep analysis of space-time, a 
conceptual analysis maybe analogous to the one done by the Viennese 
physicist-philosopher Ernst Mach who kicked Newton’s absolute space and 
absolute time form their throne. The hope is that in the end we will have new 
physics analogous to Einstein’s new physics in the two theories of relativity.”
 

-- 
You received this message because you are subscribed to the Google Groups 
"Everything List" group.
To unsubscribe from this group and stop receiving emails from it, send an email 
to everything-list+unsubscr...@googlegroups.com.
To view this discussion on the web visit 
https://groups.google.com/d/msgid/everything-list/243069131.8543635.1693828098555%40mail1.libero.it.


Re: Is Many Worlds Falsifiable?

2023-09-04 Thread Bruce Kellett
On Mon, Sep 4, 2023 at 7:15 PM 'scerir' via Everything List <
everything-list@googlegroups.com> wrote:

> *local, non-local, separable, non-separable, causes, correlations,
> influences, physical speed limit, speed of quantum influences, space-time,
> out of space-time, many worlds, many physical worlds, what a mess* Testing
> spooky action at a distance
> D. Salart
> , A.
> Baas 
> , C. Branciard
> 
> , N. Gisin
> , H.
> Zbinden
> 
>
> In science, one observes correlations and invents theoretical models that
> describe them. In all sciences, besides quantum physics, all correlations
> are described by either of two mechanisms. Either a first event influences
> a second one by sending some information encoded in bosons or molecules or
> other physical carriers, depending on the particular science. Or the
> correlated events have some common causes in their common past.
> Interestingly, quantum physics predicts an entirely different kind of cause
> for some correlations, named entanglement. This new kind of cause reveals
> itself, e.g., in correlations that violate Bell inequalities (hence cannot
> be described by common causes) between space-like separated events (hence
> cannot be described by classical communication). Einstein branded it as
> spooky action at a distance. A real spooky action at a distance would
> require a faster than light influence defined in some hypothetical
> universally privileged reference frame. Here we put stringent experimental
> bounds on the speed of all such hypothetical influences. We performed a
> Bell test during more than 24 hours between two villages separated by 18 km
> and approximately east-west oriented, with the source located precisely in
> the middle. We continuously observed 2-photon interferences well above the
> Bell inequality threshold. Taking advantage of the Earth's rotation, the
> configuration of our experiment allowed us to determine, for any
> hypothetically privileged frame, a lower bound for the speed of this spooky
> influence. For instance, if such a privileged reference frame exists and is
> such that the Earth's speed in this frame is less than 10^-3 that of the
> speed of light, then the speed of this spooky influence would have to
> exceed that of light by at least 4 orders of magnitude.
>
> Comments: Preliminary version of Nature 454, 861-864 (14 August 2008). 5
> pages and 5 figures
> Subjects: Quantum Physics (quant-ph)
> Cite as: arXiv:0808.3316  [quant-ph]
>   (or arXiv:0808.3316v1  [quant-ph] for
> this version)
>   https://doi.org/10.48550/arXiv.0808.3316
> Journal reference: Nature 454, 861-864 (14 August 2008)
> Related DOI: https://doi.org/10.1038/nature07121
>

Interesting. But I don't think non-locality is to be understood in these
terms: It probably involves rethinking our notions of time and space.

Bruce

-- 
You received this message because you are subscribed to the Google Groups 
"Everything List" group.
To unsubscribe from this group and stop receiving emails from it, send an email 
to everything-list+unsubscr...@googlegroups.com.
To view this discussion on the web visit 
https://groups.google.com/d/msgid/everything-list/CAFxXSLRFqQTt5fCf-jQfw9y6%3DNk3upv0CE88vm0yh-tZSFdHwg%40mail.gmail.com.


Re: Is Many Worlds Falsifiable?

2023-09-04 Thread Bruce Kellett
On Mon, Sep 4, 2023 at 5:41 PM John Clark  wrote:

> On Sun, Sep 3, 2023 at 7:54 PM Bruce Kellett 
> wrote:
>
> *> Special relativity merely forbids the transmission of anything
>> 'physical' faster than light (FTL). It is easily possible to transfer
>> information FTL.*
>>
>
> *BULLSHIT!*
>
> * > Consider the following. shine a laser at the moon, then scan across
>> the surface of the moon. The spot of light on the moon's surface clearly
>> can move at any speed, particularly FTL. Now, if you use the laser to
>> transmit a message to the first point, then scan away and re-transmit to
>> the second location, you can certainly transmit information FTL.*
>>
>
> *Don't be ridiculous! Light takes about 1 1/4 seconds to reach the Moon,
> if I  aim a laser at point X on the Moon and then move it to point Y also
> on the Moon it will take the usual 1 1/4 seconds after I moved my laser
> before anybody at point X observes that the light coming from Earth has
> gone off, and it will take the usual 1 1/4 seconds before anybody at point
> Y sees a light from Earth go on, and 2 1/2 seconds before anybody on planet
> Earth sees the spot of light at point X start to move. Nobody on the Earth
> or on the Moon has received or transmitted any information faster than
> light. If it was possible to transmit information FTL according to
> relativity you could send a message into the past, you could talk to  the
> Bruce Kellett of yesterday and that would create paradoxes.*
>

No. The example was not particularly well thought out. My point is that
geometrical motions can exceed light velocity, and distant galaxies recede
at greater than light speed. Light speed limits only physical transmission,
unless by tachyons. In fine, understanding non-locality probably involves
refining our understanding of space and time.

*> "Non-local" does no mean that anything physical is transmitted FTL.*
>>
>
>
> *Being "local" means that there is a finite limit to the speed of
> PHYSICAL causality, and in this universe that speed seems to be the speed
> of light. *
>

Things do need to be rethought in the light of violations of the Bell
inequalities and the unambiguous non-locality that this implies.


> *>> What in the multiverse are you talking about?!  If Many Worlds is
>>> correct then if "you" (personal pronouns can become problematic when
>>> talking about the multiverse) perform the polarizer experiment on 1 million
>>> entangled photons then in the multiverse there are 1 million new Bruce
>>> Kelletts that are absolutely identical in every way EXCEPT for the fact
>>> that they each have 1 million different memories of how those 1 million
>>> entangle protons behaved when they hit their polarizers.*
>>>
>>
But for any one observer, even in many worlds, there is only ever one
outcome for each experiment. And the existence of other words does not
affect the result that that individual observer obtains. Hence Bell's
theorem applies separately for every individual, even in many worlds.


*> There may well be copies of the experimenter in MWI, but for any
>> particular individual among these copies, the outcome of their experiments
>> are unique.*
>>
>
> *Yes.*
>
>
>> *> Bell's theorem applies equally to all the copies individually.*
>>
>
> *Yes, and in all of them all the Bruce Kelletts can experimentally confirm
> that Bell's Inequality can be violated which would be logically impossible
> if things were both realistic and local. *
>

That dichotomy does not apply.

*>> Entangled photons have opposite polarizations so if an entangled photon
>>> of undetermined polarization hits a polarizer oriented in the up" direction
>>> (what you call "up" could be any direction) and Many Worlds is correct then
>>> the universe splits many times but in NO universe is there a case where 2
>>> entangle photons both make it through polarizers oriented in the same
>>> direction.*
>>
>>
That is one of the things that have to be explained.


*> Mere assertion is not proof of anything.*
>>
>
> *DO YOUR HOMEWORK! It's been known for hundreds of years that light beams
> with opposite polarizations treat polarizers in opposite ways, and it's
> been known since 1905 that light beams are made up of photons. None of this
> is controversial, it's physics 101. *
>

So how do entangled photons end up with opposite polarizations in an
arbitrarily chosen direction?

Bruce

-- 
You received this message because you are subscribed to the Google Groups 
"Everything List" group.
To unsubscribe from this group and stop receiving emails from it, send an email 
to everything-list+unsubscr...@googlegroups.com.
To view this discussion on the web visit 
https://groups.google.com/d/msgid/everything-list/CAFxXSLQBaegzercMbw_mV1dU3MYQX3XH4Lj8BYKDqVqf8AFAFQ%40mail.gmail.com.


Re: Is Many Worlds Falsifiable?

2023-09-04 Thread 'scerir' via Everything List
local, non-local, separable, non-separable, causes, correlations, influences, 
physical speed limit, speed of quantum influences, space-time, out of 
space-time, many worlds, many physical worlds, what a mess


Testing spooky action at a distance


D. Salart 
https://arxiv.org/search/quant-ph?searchtype=author&query=Salart%2C+D, A. Baas 
https://arxiv.org/search/quant-ph?searchtype=author&query=Baas%2C+A, C. 
Branciard 
https://arxiv.org/search/quant-ph?searchtype=author&query=Branciard%2C+C, N. 
Gisin https://arxiv.org/search/quant-ph?searchtype=author&query=Gisin%2C+N, H. 
Zbinden https://arxiv.org/search/quant-ph?searchtype=author&query=Zbinden%2C+H

> In science, one observes correlations and invents theoretical models that 
> describe them. In all sciences, besides quantum physics, all correlations are 
> described by either of two mechanisms. Either a first event influences a 
> second one by sending some information encoded in bosons or molecules or 
> other physical carriers, depending on the particular science. Or the 
> correlated events have some common causes in their common past. 
> Interestingly, quantum physics predicts an entirely different kind of cause 
> for some correlations, named entanglement. This new kind of cause reveals 
> itself, e.g., in correlations that violate Bell inequalities (hence cannot be 
> described by common causes) between space-like separated events (hence cannot 
> be described by classical communication). Einstein branded it as spooky 
> action at a distance. A real spooky action at a distance would require a 
> faster than light influence defined in some hypothetical universally 
> privileged reference frame. Here we put stringent experimental bounds on the 
> speed of all such hypothetical influences. We performed a Bell test during 
> more than 24 hours between two villages separated by 18 km and approximately 
> east-west oriented, with the source located precisely in the middle. We 
> continuously observed 2-photon interferences well above the Bell inequality 
> threshold. Taking advantage of the Earth's rotation, the configuration of our 
> experiment allowed us to determine, for any hypothetically privileged frame, 
> a lower bound for the speed of this spooky influence. For instance, if such a 
> privileged reference frame exists and is such that the Earth's speed in this 
> frame is less than 10^-3 that of the speed of light, then the speed of this 
> spooky influence would have to exceed that of light by at least 4 orders of 
> magnitude.
> 
Comments:   Preliminary version of Nature 454, 861-864 (14 August 2008). 5 
pages and 5 figures
Subjects:   Quantum Physics (quant-ph)
Cite as:arXiv:0808.3316 https://arxiv.org/abs/0808.3316 [quant-ph]
(or arXiv:0808.3316v1 https://arxiv.org/abs/0808.3316v1 [quant-ph] for 
this version)
https://doi.org/10.48550/arXiv.0808.3316
Journal reference:  Nature 454, 861-864 (14 August 2008)
Related DOI:https://doi.org/10.1038/nature07121


 

-- 
You received this message because you are subscribed to the Google Groups 
"Everything List" group.
To unsubscribe from this group and stop receiving emails from it, send an email 
to everything-list+unsubscr...@googlegroups.com.
To view this discussion on the web visit 
https://groups.google.com/d/msgid/everything-list/1481523919.8569752.1693818953318%40mail1.libero.it.


Re: Is Many Worlds Falsifiable?

2023-09-04 Thread John Clark
On Sun, Sep 3, 2023 at 7:54 PM Bruce Kellett  wrote:

*> Special relativity merely forbids the transmission of anything
> 'physical' faster than light (FTL). It is easily possible to transfer
> information FTL.*
>

*BULLSHIT!*

* > Consider the following. shine a laser at the moon, then scan across the
> surface of the moon. The spot of light on the moon's surface clearly can
> move at any speed, particularly FTL. Now, if you use the laser to transmit
> a message to the first point, then scan away and re-transmit to the second
> location, you can certainly transmit information FTL.*
>


*Don't be ridiculous! Light takes about 1 1/4 seconds to reach the Moon, if
I  aim a laser at point X on the Moon and then move it to point Y also on
the Moon it will take the usual 1 1/4 seconds after I moved my laser before
anybody at point X observes that the light coming from Earth has gone off,
and it will take the usual 1 1/4 seconds before anybody at point Y sees a
light from Earth go on, and 2 1/2 seconds before anybody on planet Earth
sees the spot of light at point X start to move. Nobody on the Earth or on
the Moon has received or transmitted any information faster than light. If
it was possible to transmit information FTL according to relativity you
could send a message into the past, you could talk to  the Bruce Kellett of
yesterday and that would create paradoxes.*

> *> "Non-local" does no mean that anything physical is transmitted FTL.*
>

*Being "local" means that there is a finite limit to the speed of
PHYSICAL causality, and in this universe that speed seems to be the speed
of light. *


*>> What in the multiverse are you talking about?!  If Many Worlds is
>> correct then if "you" (personal pronouns can become problematic when
>> talking about the multiverse) perform the polarizer experiment on 1 million
>> entangled photons then in the multiverse there are 1 million new Bruce
>> Kelletts that are absolutely identical in every way EXCEPT for the fact
>> that they each have 1 million different memories of how those 1 million
>> entangle protons behaved when they hit their polarizers.*
>>
>
> *> There may well be copies of the experimenter in MWI, but for any
> particular individual among these copies, the outcome of their experiments
> are unique.*
>

*Yes.*


> *> Bell's theorem applies equally to all the copies individually.*
>

*Yes, and in all of them all the Bruce Kelletts can experimentally confirm
that Bell's Inequality can be violated which would be logically impossible
if things were both realistic and local. *

*>> Entangled photons have opposite polarizations so if an entangled photon
>> of undetermined polarization hits a polarizer oriented in the up" direction
>> (what you call "up" could be any direction) and Many Worlds is correct then
>> the universe splits many times but in NO universe is there a case where 2
>> entangle photons both make it through polarizers oriented in the same
>> direction.*
>
>
> *> Mere assertion is not proof of anything.*
>

*DO YOUR HOMEWORK! It's been known for hundreds of years that light beams
with opposite polarizations treat polarizers in opposite ways, and it's
been known since 1905 that light beams are made up of photons. None of this
is controversial, it's physics 101. *

  John K ClarkSee what's on my new list at  Extropolis

p1o

-- 
You received this message because you are subscribed to the Google Groups 
"Everything List" group.
To unsubscribe from this group and stop receiving emails from it, send an email 
to everything-list+unsubscr...@googlegroups.com.
To view this discussion on the web visit 
https://groups.google.com/d/msgid/everything-list/CAJPayv2QS05g1xrJh2-NFHcyFECHd0K2T95xPd%3DrzJbmy%3DngAg%40mail.gmail.com.


Re: Is Many Worlds Falsifiable?

2023-09-03 Thread Bruce Kellett
On Sun, Sep 3, 2023 at 9:58 PM John Clark  wrote:

> On Sun, Sep 3, 2023 at 3:43 AM Bruce Kellett 
> wrote:
>
> *> You appear to agree that Bell's theorem, given its assumptions, shows
>> that no local hidden variable account of these correlations is possible.*
>>
>
> *Of course I agree with Bell's theorem, if I disagreed I would in effect
> be saying that high school algebra was wrong.  *
>
> *> You then expect at least one of two things must be true:*
>> *1) The universe is not realistic.*
>> *2)The universe is non-local.*
>> *It is not clear how you get to this dichotomy,*
>>
>
> *I don't see anything unclear about it. If 2 entangled photons can
> exchange information faster than light*
>

Special relativity merely forbids the transmission of anything 'physical'
faster than light (FTL). It is easily possible to transfer information FTL.
Consider the following. shine a laser at the moon, then scan across the
surface of the moon. The spot of light on the moon's surface clearly can
move at any speed, particularly FTL. Now, if you use the laser to transmit
a message to the first point, then scan away and re-transmit to the second
location, you can certainly transmit information FTL.


*HOLD ON! Before you start talking about "another objection" explain the
> first one. Please explain how Hugh Everett's theory allows for the
> communication of information faster than the speed of light.*
>

"Non-local" does no mean that anything physical is transmitted FTL.


*> He claims that many worlds invalidates Bell's assumption that
>> experiments have just one outcome. But in that whole history of physics,
>> that has always been true.There has never been a case in which an
>> experimenter has seen more than one outcome in a single experiment. Bell's
>> theorem applies in many worlds exactly as it applies in single world
>> theories. The reason is that when Alice and Bob perform a series of
>> polarization measurements on entangled particles to ascertain the
>> correlation, all their measurements and calculations take place in a single
>> world. In no case do they see more than a single result for each
>> measurement,*
>>
>
> *What in the multiverse are you talking about?!  If Many Worlds is correct
> then if "you" (personal pronouns can become problematic when talking about
> the multiverse) perform the polarizer experiment on 1 million entangled
> photons then in the multiverse there are 1 million new Bruce Kelletts that
> are absolutely identical in every way EXCEPT for the fact that they each
> have 1 million different memories of how those 1 million entangle protons
> behaved when they hit their polarizers.*
>

There may well be copies of the experimenter in MWI, but for any particular
individual among these copies, the outcome of their experiments are unique.
Bell's theorem applies equally to all the copies individually.


*> f you disagree with this argument, then I invite you to provide a
> counterexample by providing a local account of the correlations.*
>

*OK. Entangled photons have opposite polarizations so if an entangled
photon of undetermined polarization hits a polarizer oriented in the up"
direction (what you call "up" could be any direction) and Many Worlds is
correct then the universe splits many times but in NO universe is there a
case where 2 entangle photons both make it through polarizers oriented in
the same direction.*

*Mere assertion is not proof of anything.*

*Bruce*

-- 
You received this message because you are subscribed to the Google Groups 
"Everything List" group.
To unsubscribe from this group and stop receiving emails from it, send an email 
to everything-list+unsubscr...@googlegroups.com.
To view this discussion on the web visit 
https://groups.google.com/d/msgid/everything-list/CAFxXSLTR_zqxVyFqkNmpzgpfm21sARuicie_QyjTrdUCxXHnhw%40mail.gmail.com.


Re: Is Many Worlds Falsifiable?

2023-09-03 Thread Bruce Kellett
On Sun, Sep 3, 2023 at 11:37 AM smitra  wrote:

> On 31-08-2023 06:08, Bruce Kellett wrote:
>
> > That is all very well, but it is not a local account of violations of
> > the Bell inequalities. You merely claim that the local theory is such
> > an account, but you do not spell it out.
>
> John has addressed this in a subsequent reply where he cites an old
> reply giving the detailed account involving polarizers.
>

I have responded to John in a separate post. He appears to have a very weak
grasp of logic, and his arguments are not valid.


Thing is that in conventional QM we only have the dynamics only involves
> the Schrödinger equation and collapse.


The Schrodinger equation is not necessary for quantum mechanics. The
Heiseberg matrix formulation does not involve the SE. Time evolution is
just a unitary transformation after all. The wave function is not
necessary. Dirac, in his book on quantum mechanics, mentions the wave
function only in an inconsequential footnote.

The time evolution according to
> the Schrödinger equation is manifestly local,


But unitary evolution according to the SE cannot account for the
correlation of entangled particles.

while the collapse is the
> only non-local part. So, any version of QM in which there is no collapse
> is guaranteed to be local.
>

> Another important thing to note here is that Bell's theorem only applies
> to hidden variable theories, it does not apply to QM in general.


Where on earth did you get that idea from? As John has pointed out, Bell's
theorem does not require even quantum mechanics. It is just a piece of
mathematics.It applies with complete generality to quantum mechanics, with
or without hidden variables.



> The MWI
> is not a hidden variables theory, so Bell's theorem has nothing
> whatsoever to say about this.
>

Again, As I pointed out to John, even if you assume that Bell's theorem
does not apply to MWI (and of course it does), then it does not follow that
the theory is local. It could be non-local for reasons unconnected with
Bell's theorem.


> We have had this discussion before, and you couldn't give the detailed
> > local account then either.
>
> You disputed the well established fact that all known interactions are
> locaThat is not a well establised fact. Given the violations of the Bell
> inequalitiers, the only well established fact is that standard QM is
> non-local.




> You would not take a formal answer like
>
>   psi(x, t) = Exp(-i H/hbar t) psi(x,0)
>
> where H is the a local Hamiltonian that describes the dynamics for an
> answer.


Of course that is not an answer. It is merely a re-stating of your
contention that QM is always local. Whether or not that Hamiltonian
formulation is able to account for the Bell-type correlations is precisely
the point at issue. Restating that the correlations do indeed have a local
explanation does not take us any further forward.

You wanted me two explicitly write out H for a Bell-type
> experiment for H a manifestly local Hamiltonian, and then to compute the
> time evolution. Me not doing that was your argument that something
> non-local was going on here.
>

No. My argument hinges on the applicability and universality of Bell's
theorem. Your failure to provide a counterexample was merely proof that you
don't know what you are talking about.

 Bell's theorem applies in Everettian
> > quantum mechanics in exactly the same way as it applies in one-world
> > accounts. Bell's theorem proves that the effect is non-local, so no
> > local account is possible in any interpretation of QM.
>
> Bell's theorem only applies to hidden variable theories,


Bullshit. We have disposed of that canard already.

MWI is not a
> hidden variables theory. Bell's theorem does not even prove that
> Bell-type correlations are non-local in one-world interpretations of QM.
> Until that time one postulates hidden variables, Bell's theorem has
> nothing whatsoever to say about this.
>

Even if Bell's theorem does not apply, there is no reason to suppose that
the theory is local, since no classical account of the correlations is
possible.

Bruce

-- 
You received this message because you are subscribed to the Google Groups 
"Everything List" group.
To unsubscribe from this group and stop receiving emails from it, send an email 
to everything-list+unsubscr...@googlegroups.com.
To view this discussion on the web visit 
https://groups.google.com/d/msgid/everything-list/CAFxXSLQEc_i5Yw_82vZuN36XifrN0Y7F%2B0QogqyZu5gWv0VPwQ%40mail.gmail.com.


Re: Is Many Worlds Falsifiable?

2023-09-03 Thread John Clark
On Sun, Sep 3, 2023 at 3:43 AM Bruce Kellett  wrote:

*> You appear to agree that Bell's theorem, given its assumptions, shows
> that no local hidden variable account of these correlations is possible.*
>

*Of course I agree with Bell's theorem, if I disagreed I would in effect be
saying that high school algebra was wrong.  *

*> You then expect at least one of two things must be true:*
> *1) The universe is not realistic.*
> *2)The universe is non-local.*
> *It is not clear how you get to this dichotomy,*
>

*I don't see anything unclear about it. If 2 entangled photons can exchange
information faster than light then the fact that the two seem to be able to
communicate with each other that fast is no longer a mystery. And if things
(and that includes "you"), can exist in more than one state then it is no
longer a mystery that a "you" exists in one of those states.  Please note
that I did say "at least", things could be both non-local and not
realistic, in fact I wouldn't be terribly surprised if that turned out to
be the case.  *

* > but once you have it, you claim that MWI is non-realistic,..., so it
> has no need to resort to any of these non-local influences to explain
> experimental results. This conclusion is flatly illogical. Accepting one
> arm of the dichotomy does not mean that the other is false -- both could be
> false, or both could be true.*
>

*No, we have experimental proof that they both cannot be true, but yes both
could be false.  *

*> I said that realism has nothing to do with the argument over Bell
> inequalities. It simply serves to point out that ordinary one-world QM is
> also non-realistic in your sense. So it is not a special feature of many
> worlds.*
>

*I never *claimed that *experimental violation of* Bell's Inequality proved
that Many Worlds is true, I said that your original statement  "*The many
worlds idea has already been falsified because it cannot account for the
observed violation of the Bell inequalities for entangled particle**s**" is
DEAD WRONG." **I'm not certain that Many Worlds is correct, but I am
certain it's the least bad explanation anybody has come up with, at least
so far, as to why the quantum world is so weird. I'm certain of one other
thing, whatever the truth turns out to be it's going to be odd, very very
odd. Maybe Many Worlds is odd enough to be true, maybe not.*

*> MWI could be non-local for reasons unconnected with Bell's theorem.
> Arguing that Bell's theorem does not apply does not guarantee that your
> theory is local. Many people have tried this argument, but it is patently
> invalid. There is another objection to Sean's argument.*
>

*HOLD ON! Before you start talking about "another objection" explain the
first one. Please explain how Hugh Everett's theory allows for the
communication of information faster than the speed of light. *


> *> He claims that many worlds invalidates Bell's assumption that
> experiments have just one outcome. But in that whole history of physics,
> that has always been true.There has never been a case in which an
> experimenter has seen more than one outcome in a single experiment. Bell's
> theorem applies in many worlds exactly as it applies in single world
> theories. The reason is that when Alice and Bob perfore a series of
> polarization measurements on entangled particles to ascertain the
> correlation, all their measurements and calculations take place in a single
> world. In no case do they see more than a single result for each
> measurement,*
>

*What in the multiverse are you talking about?!  If Many Worlds is correct
then if "you" (personal pronouns can become problematic when talking about
the multiverse) perform the polarizer experiment on 1 million entangled
photons then in the multiverse there are 1 million new Bruce Kelletts that
are absolutely identical in every way EXCEPT for the fact that they each
have 1 million different memories of how those 1 million entangle protons
behaved when they hit their polarizers.  *


> *> f you disagree with this argument, then I invite you to provide a
> counterexample by providing a local account of the correlations.*
>

*OK. Entangled photons have opposite polarizations so if an entangled
photon of undetermined polarization hits a polarizer oriented in the up"
direction (what you call "up" could be any direction) and Many Worlds is
correct then the universe splits many times but in NO universe is there a
case where 2 entangle photons both make it through polarizers oriented in
the same direction. One example of a universe that DOES exist is one
universe where the photon here on Earth makes it through its polarizer but
its brother photon 2 million light years away in the Andromeda galaxy does
not make it through its polarizer that is also oriented in the "up"
direction. You can say that the split happens instantaneously when the
earthly photon encounters its polarizer, or you could say the split starts
on Earth and spreads outward at a finite speed, the speed o

Re: Is Many Worlds Falsifiable?

2023-09-03 Thread Bruce Kellett
On Fri, Sep 1, 2023 at 10:26 PM John Clark  wrote:

> On Thu, Aug 31, 2023 at 6:29 PM Bruce Kellett 
> wrote:
>
> *> OK. So spell out your non-realist, but local, many worlds account of
>> the violations of the Bell inequalities. It seems that you want it both
>> ways -- Bell's theorem says that MWI must be non-local, but you claim that
>> it is local? "Realism" has nothing to do with it.*
>
>
>
> "Realism" has* EVERYTHING* to do with it, and I spelled out exactly why
> in a post on May 4 2022 when somebody said they wanted to hear all the gory
> details and this is what I said:
>

I am not complaining that your explanations are too short, or too long. I
am complaining because they do not answer the question I posed: "Spell out
your non-realist, but local, many worlds account of the violations of the
Bell inequalities."

Let me summarize your argument. You appear to agree that Bell's theorem,
given its assumptions, shows that no local hidden variable account of these
correlations is possible. You then expect at least one of two things must
be true:

1) The universe is not realistic.
2)The universe is non-local.

It is not clear how you get to this dichotomy, but once you have it, you
claim that MWI is non-realistic,..., so it has no need to resort to any of
these non-local influences to explain experimental results. This conclusion
is flatly illogical. Accepting one arm of the dichotomy does not mean that
the other is false -- both could be false, or both could be true. In other
words, the theory could be both non-realistic AND non-local.

I said that realism has nothing to do with the argument over Bell
inequalities. It simply serves to point out that ordinary one-world QM is
also non-realistic in your sense. So it is not a special feature of many
worlds. Since everything in QM is non-realistic, "realism" has no
particular bearing on the violations of Bell inequalities. Your initial
dichotomy is, therefore, meaningless.

In his book "Something Deeply Hidden", Sean Carroll gives a better version
of a similar argument (p.102ff). The argument still fails, as we shall see,
but let's examine it further. Sean accepts that what Bell showed was that,
under certain superficially reasonable assumptions, the quantum mechanical
predictions are impossible to reproduce in any local theory. The
assumptions that Carroll points to are that the experimenters are free to
choose what measurement to make (no superdeterminism!) and, secondly, that
measurements have definite outcomes. He rejects this latter assumption as
being untrue in Everettian theories because all possible outcomes are
realized in the branches of the wave function, The universe as a whole
doesn't have any single outcome for a measurement: it has multiple ones. In
rejecting this supposed assumption of Bell's, Sean goes on to argue that
Bell's theorem is simply irrelevant for MWI -- it doesn't apply. Because He
has removed Bell's theorem from consideration, Sean then concludes that MWI
is local.

But, once again, this conclusion does not follow. MWI could be non-local
for reasons unconnected with Bell's theorem. Arguing that Bell's theorem
does not apply does not guarantee that your theory is local. Many people
have tried this argument, but it is patently invalid.

There is another objection to Sean's argument. He claims that many worlds
invalidates Bell's assumption that experiments have just one outcome. But
in that whole history of physics, that has always been true. There has
never been a case in which an experimenter has seen more than one outcome
in a single experiment. Bell's theorem applies in many worlds exactly as it
applies in single world theories. The reason is that when Alice and Bob
perfore a series of polarization measurements on entangled particles to
ascertain the correlation, all their measurements and calculations take
place in a single world. In no case do they see more than a single result
for each measurement, and in order to maintain agreement with universal
physical observations, the alternative outcomes postulated by MWI must
occur in separate, disjoint worlds. These 'other worlds' cannot impinge on
either Alice or Bob, or their calculations, So even if Bell's theorem does
assume a single outcome and a single world, that is all we ever have, even
in MWI. So Bell's theorem applies in full force in MWI as in the single
world case. Consequently, the correlations that Alice and Bob observe can
have no local (or local hidden variable) explanation.

If you disagree with this argument, then I invite you to provide a
counterexample by providing a local account of the correlations.

Bruce


" If you want all the details this is going to be a long post, you asked
> for it. First I'm gonna have to show that any theory (except for
> superdeterminism which is idiotic) that is deterministic, local and
> realistic cannot possibly explain the violation of Bell's Inequality that
> we see in our experiments, and then show why *a theory like Many World

Re: Is Many Worlds Falsifiable?

2023-09-02 Thread smitra

On 31-08-2023 06:08, Bruce Kellett wrote:

On Thu, Aug 31, 2023 at 12:27 PM smitra  wrote:


There is no problem here because in practice MWI is nothing more
than
the usual QM formalism to compute the outcome of experiments where
you
then assume that the ensemble of all possible outcomes really
exists.
Locality then follows from the fact hat the dynamics of the theory
is
manifestly local. The Hamiltonian only includes local interactions
and
observers are part of this dynamics. Although observer are not
explicitly treated as being part of the wavefunction that describes
the
entire system, the assumption is that in principle, this is the
case. In
practice, one can then proceed according to the usual QM formalism.


That is all very well, but it is not a local account of violations of
the Bell inequalities. You merely claim that the local theory is such
an account, but you do not spell it out.


John has addressed this in a subsequent reply where he cites an old 
reply giving the detailed account involving polarizers.



Thing is that in conventional QM we only have the dynamics only involves 
the Schrödinger equation and collapse. The time evolution according to 
the Schrödinger equation is manifestly local, while the collapse is the 
only non-local part. So, any version of QM in which there is no collapse 
is guaranteed to be local.


Another important thing to note here is that Bell's theorem only applies 
to hidden variable theories, it does not apply to QM in general. The MWI 
is not a hidden variables theory, so Bell's theorem has nothing 
whatsoever to say about this.




We have had this discussion before, and you couldn't give the detailed
local account then either.


You disputed the well established fact that all known interactions are 
local. You would not take a formal answer like


 psi(x, t) = Exp(-i H/hbar t) psi(x,0)

where H is the a local Hamiltonian that describes the dynamics for an 
answer.  You wanted me two explicitly write out H for a Bell-type 
experiment for H a manifestly local Hamiltonian, and then to compute the 
time evolution. Me not doing that was your argument that something 
non-local was going on here.


 Bell'e theorem applies in Everettian

quantum mechanics in exactly the same way as it applies in one-world
accounts. Bell's theorem proves that the effect is non-local, so no
local account is possible in any interpretation of QM.


Bell's theorem only applies to hidden variable theories, MWI is not a 
hidden variables theory. Bell's theorem does not even prove that 
Bell-type correlations are non-local in one-world interpretations of QM. 
Until that time one postulates hidden variables, Bell's theorem has 
nothing whatsoever to say about this.


Saibal








John points out the thought experiments by Deutsch makes it clear
that
the usual QM formalism will not work in certain cases, that will
then
falsify the ad hoc collapse postulate. If you then believe that MWI
cannot account for violation of Bell's inequalities while ordinary
QM
can, then that begs the question of how removing the FAPP
unobservable
sectors where all other outcomes are realized, could matter at all.


The other sectors are not just FAPP unoservable, they are not
observable in principle. How could the presence of unobservable fairy
tales affect anything at all? The standard account of violations of
the Bell inequalities in quantum mechanics relies on the notion of
non-locality. And since the effect is non-local, no local account is
possible.

Bruce

 --
You received this message because you are subscribed to the Google
Groups "Everything List" group.
To unsubscribe from this group and stop receiving emails from it, send
an email to everything-list+unsubscr...@googlegroups.com.
To view this discussion on the web visit
https://groups.google.com/d/msgid/everything-list/CAFxXSLTAg%3D%2BVf34FWWYXZ3%2BxKwkZmNWdt2VM_CRfHHBm3nqE%3Dg%40mail.gmail.com
[1].


Links:
--
[1]
https://groups.google.com/d/msgid/everything-list/CAFxXSLTAg%3D%2BVf34FWWYXZ3%2BxKwkZmNWdt2VM_CRfHHBm3nqE%3Dg%40mail.gmail.com?utm_medium=email&utm_source=footer


--
You received this message because you are subscribed to the Google Groups 
"Everything List" group.
To unsubscribe from this group and stop receiving emails from it, send an email 
to everything-list+unsubscr...@googlegroups.com.
To view this discussion on the web visit 
https://groups.google.com/d/msgid/everything-list/4e601eea2b01d2f7c4bb5efe23ba904b%40zonnet.nl.


Re: Is Many Worlds Falsifiable?

2023-09-01 Thread Stathis Papaioannou
On Sat, 2 Sep 2023 at 06:20, Jesse Mazer  wrote:

> I also think superdeterminism is "local" only on a technicality. If one is
> looking at the general class of superdeterminist theories rather than just
> the specific subset designed to reproduce quantum mechanical statistics,
> one could easily come up with a superdeterminist theory that allowed for
> apparent FTL information transmission, where for example everytime a
> "transmitter" was wiggled a certain way by experimenters, a corresponding
> "receiver" at a spacelike separation would always wiggle the same way. The
> superdeterminist "explanation" for this could be that while the dynamical
> laws obey locality, the match between transmitter and receiver is simply
> ensured by a special choice of initial conditions at the Big Bang (one that
> requires a specific kind of match between events in the past light cone of
> the reception-events that are not in the past light cone of the
> transmission-events, and events in the past light cone of the
> transmission-events that are not in the past light cone of the
> reception-events).
>

Doesn’t ordinary determinism make this logically possible, since all
information is effectively produced by the same pseudorandom number
generator? It isn’t actually the case in the universe we live in, because
that’s the nature of the algorithm.


> On Fri, Sep 1, 2023 at 2:50 PM Jason Resch  wrote:
>
>> I agree with John. What makes superdeterminism weird isn't the
>> determinism part. It's that the system is also rigged against us to produce
>> the Bell inequality.
>>
>> I am not sure if you saw my recent example on extropy-chat with flipping
>> coins and always seeing heads 66% of the time, no matter what we do, but
>> superdeterminism is basically saying that's just how it is the universe has
>> preordained that humans flip coins such that they come up head's 66% of the
>> time.
>>
>> Jason
>>
>> Jason
>>
>> On Fri, Sep 1, 2023, 2:47 PM Stathis Papaioannou 
>> wrote:
>>
>>>
>>>
>>> On Sat, 2 Sep 2023 at 04:20, John Clark  wrote:
>>>
 On Fri, Sep 1, 2023 at 1:22 PM Stathis Papaioannou 
 wrote:

  >> according to superdeterminism the particular initial condition the
>> universe was in 13.8 billion years ago has determined if you think
>> superdeterminism is a reasonable theory or if you think it's complete
>> bullshit. As for me I was determined to believe it's bullshit.
>>
>
> *>I still struggle to see the difference between determinism and
> superdeterminism. They both say that there is no true randomness*
>

 Yes.


> * > which includes randomness in how the experimenters set up their
> experiment.*
>

 No. Knowing the laws of physics is not enough, to make predictions you
 also need to know the initial conditions. Superdeterminism says more than a
 given state of the universe is the mathematical product of the previous
 state, superdeterminism assumes, for no particular reason, that out of the
 infinite number of states the universe could've started out at, 13.8
 billion years ago it was in the one and only one particular state that
 would make experimenters 13.8 billion years later "choose" to set their
 instruments in such a way that they always *INCORRECTLY* conclude that
 things can *NOT* be both realistic and local. It would be absolutely
 impossible to make a larger assumption than this, and that is why it is the
 largest violation of Occam's Razor conceivable. There are an infinite
 number of initial conditions the universe could've started out in and in
 which things would be deterministic today, but one and only one initial
 condition would produce the universe in which superdeterminism is true. And
 if superdeterminism were true then there would be no point in performing
 scientific experiments since there would be no reason for them to lead
 to the truth, and yet airplanes fly and bridges don't collapse so they do
 seem to lead to the truth, there is no way to explain that unless the
 initial conditions were even further restrained such that we set our
 instruments correctly on all experiments *EXCEPT* when the
 experimenters try to test for realism or locality, then we "choose" to set
 them incorrectly. That's why I don't understand how anyone can take this
 seriously. That is why I think superdeterminism is bullshit.

>>>
>>> Bell seemed to think that super determinism meant that the mind of the
>>> experimenters was determined along with everything else, which he described
>>> as a lack of “free will” (it seems he meant by this lack of randomness in
>>> their minds), which he thought was an assumption in the experiment:
>>>
>>> “There is a way to escape the inference of superluminal
>>>  speeds and spooky action
>>> at a distance. But it involves absolute determinism
>>> 

Re: Is Many Worlds Falsifiable?

2023-09-01 Thread Jesse Mazer
I also think superdeterminism is "local" only on a technicality. If one is
looking at the general class of superdeterminist theories rather than just
the specific subset designed to reproduce quantum mechanical statistics,
one could easily come up with a superdeterminist theory that allowed for
apparent FTL information transmission, where for example everytime a
"transmitter" was wiggled a certain way by experimenters, a corresponding
"receiver" at a spacelike separation would always wiggle the same way. The
superdeterminist "explanation" for this could be that while the dynamical
laws obey locality, the match between transmitter and receiver is simply
ensured by a special choice of initial conditions at the Big Bang (one that
requires a specific kind of match between events in the past light cone of
the reception-events that are not in the past light cone of the
transmission-events, and events in the past light cone of the
transmission-events that are not in the past light cone of the
reception-events).

Jesse

On Fri, Sep 1, 2023 at 2:50 PM Jason Resch  wrote:

> I agree with John. What makes superdeterminism weird isn't the determinism
> part. It's that the system is also rigged against us to produce the Bell
> inequality.
>
> I am not sure if you saw my recent example on extropy-chat with flipping
> coins and always seeing heads 66% of the time, no matter what we do, but
> superdeterminism is basically saying that's just how it is the universe has
> preordained that humans flip coins such that they come up head's 66% of the
> time.
>
> Jason
>
> Jason
>
> On Fri, Sep 1, 2023, 2:47 PM Stathis Papaioannou 
> wrote:
>
>>
>>
>> On Sat, 2 Sep 2023 at 04:20, John Clark  wrote:
>>
>>> On Fri, Sep 1, 2023 at 1:22 PM Stathis Papaioannou 
>>> wrote:
>>>
>>>  >> according to superdeterminism the particular initial condition the
> universe was in 13.8 billion years ago has determined if you think
> superdeterminism is a reasonable theory or if you think it's complete
> bullshit. As for me I was determined to believe it's bullshit.
>

 *>I still struggle to see the difference between determinism and
 superdeterminism. They both say that there is no true randomness*

>>>
>>> Yes.
>>>
>>>
 * > which includes randomness in how the experimenters set up their
 experiment.*

>>>
>>> No. Knowing the laws of physics is not enough, to make predictions you
>>> also need to know the initial conditions. Superdeterminism says more than a
>>> given state of the universe is the mathematical product of the previous
>>> state, superdeterminism assumes, for no particular reason, that out of the
>>> infinite number of states the universe could've started out at, 13.8
>>> billion years ago it was in the one and only one particular state that
>>> would make experimenters 13.8 billion years later "choose" to set their
>>> instruments in such a way that they always *INCORRECTLY* conclude that
>>> things can *NOT* be both realistic and local. It would be absolutely
>>> impossible to make a larger assumption than this, and that is why it is the
>>> largest violation of Occam's Razor conceivable. There are an infinite
>>> number of initial conditions the universe could've started out in and in
>>> which things would be deterministic today, but one and only one initial
>>> condition would produce the universe in which superdeterminism is true. And
>>> if superdeterminism were true then there would be no point in performing
>>> scientific experiments since there would be no reason for them to lead
>>> to the truth, and yet airplanes fly and bridges don't collapse so they do
>>> seem to lead to the truth, there is no way to explain that unless the
>>> initial conditions were even further restrained such that we set our
>>> instruments correctly on all experiments *EXCEPT* when the
>>> experimenters try to test for realism or locality, then we "choose" to set
>>> them incorrectly. That's why I don't understand how anyone can take this
>>> seriously. That is why I think superdeterminism is bullshit.
>>>
>>
>> Bell seemed to think that super determinism meant that the mind of the
>> experimenters was determined along with everything else, which he described
>> as a lack of “free will” (it seems he meant by this lack of randomness in
>> their minds), which he thought was an assumption in the experiment:
>>
>> “There is a way to escape the inference of superluminal
>>  speeds and spooky action at
>> a distance. But it involves absolute determinism
>>  in the universe, the
>> complete absence of free will .
>> Suppose the world is super-deterministic, with not just inanimate nature
>> running on behind-the-scenes clockwork, but with our behavior, including
>> our belief that we are free to choose to do one experiment rather than
>> another, absolutely predetermined, including the 

Re: Is Many Worlds Falsifiable?

2023-09-01 Thread John Clark
On Fri, Sep 1, 2023 at 2:50 PM Jason Resch  wrote:

*> I agree with John. What makes superdeterminism weird isn't the
> determinism part. It's that the system is also rigged against us to produce
> the Bell inequality.*
>

Yes.

*> I am not sure if you saw my recent example on extropy-chat with flipping
> coins and always seeing heads 66% of the time, no matter what we do, but
> superdeterminism is basically saying that's just how it is the universe has
> preordained that humans flip coins such that they come up head's 66% of the
> time.*
>

That's a good example of the sort of thing I was talking about,
superdeterminism claims that the universe is lying to us. It sort of
reminds me of the holy rollers and snake handlers who insist that God
buried dinosaur bones deep in the ground just 5000 years ago but made them
look like they were millions of years old in order to test our faith. God
is supposed to be much smarter and much more powerful than we are so if he
wants to fool us he certainly has the capacity to do so, but if we fall for
his trickery He will torture for an infinite (the Bible doesn't make clear
if that infinity is Aleph 0 or Aleph1) number of years. But as George
Carlin reminds us, HE LOVES YOU!

 John K ClarkSee what's on my new list at  Extropolis

ghe


>
> On Fri, Sep 1, 2023, 2:47 PM Stathis Papaioannou 
> wrote:
>
>>
>>
>> On Sat, 2 Sep 2023 at 04:20, John Clark  wrote:
>>
>>> On Fri, Sep 1, 2023 at 1:22 PM Stathis Papaioannou 
>>> wrote:
>>>
>>>  >> according to superdeterminism the particular initial condition the
> universe was in 13.8 billion years ago has determined if you think
> superdeterminism is a reasonable theory or if you think it's complete
> bullshit. As for me I was determined to believe it's bullshit.
>

 *>I still struggle to see the difference between determinism and
 superdeterminism. They both say that there is no true randomness*

>>>
>>> Yes.
>>>
>>>
 * > which includes randomness in how the experimenters set up their
 experiment.*

>>>
>>> No. Knowing the laws of physics is not enough, to make predictions you
>>> also need to know the initial conditions. Superdeterminism says more than a
>>> given state of the universe is the mathematical product of the previous
>>> state, superdeterminism assumes, for no particular reason, that out of the
>>> infinite number of states the universe could've started out at, 13.8
>>> billion years ago it was in the one and only one particular state that
>>> would make experimenters 13.8 billion years later "choose" to set their
>>> instruments in such a way that they always *INCORRECTLY* conclude that
>>> things can *NOT* be both realistic and local. It would be absolutely
>>> impossible to make a larger assumption than this, and that is why it is the
>>> largest violation of Occam's Razor conceivable. There are an infinite
>>> number of initial conditions the universe could've started out in and in
>>> which things would be deterministic today, but one and only one initial
>>> condition would produce the universe in which superdeterminism is true. And
>>> if superdeterminism were true then there would be no point in performing
>>> scientific experiments since there would be no reason for them to lead
>>> to the truth, and yet airplanes fly and bridges don't collapse so they do
>>> seem to lead to the truth, there is no way to explain that unless the
>>> initial conditions were even further restrained such that we set our
>>> instruments correctly on all experiments *EXCEPT* when the
>>> experimenters try to test for realism or locality, then we "choose" to set
>>> them incorrectly. That's why I don't understand how anyone can take this
>>> seriously. That is why I think superdeterminism is bullshit.
>>>
>>
>>

-- 
You received this message because you are subscribed to the Google Groups 
"Everything List" group.
To unsubscribe from this group and stop receiving emails from it, send an email 
to everything-list+unsubscr...@googlegroups.com.
To view this discussion on the web visit 
https://groups.google.com/d/msgid/everything-list/CAJPayv0HXrnUD5bdHmF_691jpXexWh_cKj3A%3DjRKwoT2kZCbBw%40mail.gmail.com.


Re: Is Many Worlds Falsifiable?

2023-09-01 Thread Stathis Papaioannou
On Sat, 2 Sep 2023 at 05:00, John Clark  wrote:

>
>
> On Fri, Sep 1, 2023 at 2:47 PM Stathis Papaioannou 
> wrote:
>
> >> No. Knowing the laws of physics is not enough, to make predictions you
>>> also need to know the initial conditions. Superdeterminism says more than a
>>> given state of the universe is the mathematical product of the previous
>>> state, superdeterminism assumes, for no particular reason, that out of the
>>> infinite number of states the universe could've started out at, 13.8
>>> billion years ago it was in the one and only one particular state that
>>> would make experimenters 13.8 billion years later "choose" to set their
>>> instruments in such a way that they always *INCORRECTLY* conclude that
>>> things can *NOT* be both realistic and local. It would be absolutely
>>> impossible to make a larger assumption than this, and that is why it is the
>>> largest violation of Occam's Razor conceivable. There are an infinite
>>> number of initial conditions the universe could've started out in and in
>>> which things would be deterministic today, but one and only one initial
>>> condition would produce the universe in which superdeterminism is true. And
>>> if superdeterminism were true then there would be no point in performing
>>> scientific experiments since there would be no reason for them to lead
>>> to the truth, and yet airplanes fly and bridges don't collapse so they do
>>> seem to lead to the truth, there is no way to explain that unless the
>>> initial conditions were even further restrained such that we set our
>>> instruments correctly on all experiments *EXCEPT* when the
>>> experimenters try to test for realism or locality, then we "choose" to set
>>> them incorrectly. That's why I don't understand how anyone can take this
>>> seriously. That is why I think superdeterminism is bullshit.
>>>
>>
>> *> Bell seemed to think that super determinism meant that the mind of the
>> experimenters was determined along with everything else, which he described
>> as a lack of “free will”*
>>
>
> I can't comment about that because I've never been able to figure out what
> people mean by "free will".
>
> > *it seems he meant by this lack of randomness in their minds*
>>
>
> But a lack of randomness is what you'd expect a mind to produce, at least
> a mind that was working properly, that's why when somebody does something
> we don't understand we say "why did you do that?" And if they can't give a
> good answer, a good reason, a good cause, to that then we say that their
> behavior was unreasonable.
>

Bell seems to have thought that that “free will” means true randomness in
the mind of the experimenters, and that this is an assumption in his
experiment.

>

-- 
You received this message because you are subscribed to the Google Groups 
"Everything List" group.
To unsubscribe from this group and stop receiving emails from it, send an email 
to everything-list+unsubscr...@googlegroups.com.
To view this discussion on the web visit 
https://groups.google.com/d/msgid/everything-list/CAH%3D2ypUqR%2Bo%3DY%3D3%3DqwhNV9w9fi9kjHr9Tp4DwcwYOV6A-f_h0w%40mail.gmail.com.


Re: Is Many Worlds Falsifiable?

2023-09-01 Thread John Clark
On Fri, Sep 1, 2023 at 2:47 PM Stathis Papaioannou 
wrote:

>> No. Knowing the laws of physics is not enough, to make predictions you
>> also need to know the initial conditions. Superdeterminism says more than a
>> given state of the universe is the mathematical product of the previous
>> state, superdeterminism assumes, for no particular reason, that out of the
>> infinite number of states the universe could've started out at, 13.8
>> billion years ago it was in the one and only one particular state that
>> would make experimenters 13.8 billion years later "choose" to set their
>> instruments in such a way that they always *INCORRECTLY* conclude that
>> things can *NOT* be both realistic and local. It would be absolutely
>> impossible to make a larger assumption than this, and that is why it is the
>> largest violation of Occam's Razor conceivable. There are an infinite
>> number of initial conditions the universe could've started out in and in
>> which things would be deterministic today, but one and only one initial
>> condition would produce the universe in which superdeterminism is true. And
>> if superdeterminism were true then there would be no point in performing
>> scientific experiments since there would be no reason for them to lead
>> to the truth, and yet airplanes fly and bridges don't collapse so they do
>> seem to lead to the truth, there is no way to explain that unless the
>> initial conditions were even further restrained such that we set our
>> instruments correctly on all experiments *EXCEPT* when the experimenters
>> try to test for realism or locality, then we "choose" to set them
>> incorrectly. That's why I don't understand how anyone can take this
>> seriously. That is why I think superdeterminism is bullshit.
>>
>
> *> Bell seemed to think that super determinism meant that the mind of the
> experimenters was determined along with everything else, which he described
> as a lack of “free will”*
>

I can't comment about that because I've never been able to figure out what
people mean by "free will".

> *it seems he meant by this lack of randomness in their minds*
>

But a lack of randomness is what you'd expect a mind to produce, at least a
mind that was working properly, that's why when somebody does something we
don't understand we say "why did you do that?" And if they can't give a
good answer, a good reason, a good cause, to that then we say that their
behavior was unreasonable.

 John K ClarkSee what's on my new list at  Extropolis

eep







>

-- 
You received this message because you are subscribed to the Google Groups 
"Everything List" group.
To unsubscribe from this group and stop receiving emails from it, send an email 
to everything-list+unsubscr...@googlegroups.com.
To view this discussion on the web visit 
https://groups.google.com/d/msgid/everything-list/CAJPayv1U_SL3dKT3swRHDjsnOAcq4t469VqBiLaej0Bng9ePOQ%40mail.gmail.com.


Re: Is Many Worlds Falsifiable?

2023-09-01 Thread Jason Resch
I agree with John. What makes superdeterminism weird isn't the determinism
part. It's that the system is also rigged against us to produce the Bell
inequality.

I am not sure if you saw my recent example on extropy-chat with flipping
coins and always seeing heads 66% of the time, no matter what we do, but
superdeterminism is basically saying that's just how it is the universe has
preordained that humans flip coins such that they come up head's 66% of the
time.

Jason

Jason

On Fri, Sep 1, 2023, 2:47 PM Stathis Papaioannou  wrote:

>
>
> On Sat, 2 Sep 2023 at 04:20, John Clark  wrote:
>
>> On Fri, Sep 1, 2023 at 1:22 PM Stathis Papaioannou 
>> wrote:
>>
>>  >> according to superdeterminism the particular initial condition the
 universe was in 13.8 billion years ago has determined if you think
 superdeterminism is a reasonable theory or if you think it's complete
 bullshit. As for me I was determined to believe it's bullshit.

>>>
>>> *>I still struggle to see the difference between determinism and
>>> superdeterminism. They both say that there is no true randomness*
>>>
>>
>> Yes.
>>
>>
>>> * > which includes randomness in how the experimenters set up their
>>> experiment.*
>>>
>>
>> No. Knowing the laws of physics is not enough, to make predictions you
>> also need to know the initial conditions. Superdeterminism says more than a
>> given state of the universe is the mathematical product of the previous
>> state, superdeterminism assumes, for no particular reason, that out of the
>> infinite number of states the universe could've started out at, 13.8
>> billion years ago it was in the one and only one particular state that
>> would make experimenters 13.8 billion years later "choose" to set their
>> instruments in such a way that they always *INCORRECTLY* conclude that
>> things can *NOT* be both realistic and local. It would be absolutely
>> impossible to make a larger assumption than this, and that is why it is the
>> largest violation of Occam's Razor conceivable. There are an infinite
>> number of initial conditions the universe could've started out in and in
>> which things would be deterministic today, but one and only one initial
>> condition would produce the universe in which superdeterminism is true. And
>> if superdeterminism were true then there would be no point in performing
>> scientific experiments since there would be no reason for them to lead
>> to the truth, and yet airplanes fly and bridges don't collapse so they do
>> seem to lead to the truth, there is no way to explain that unless the
>> initial conditions were even further restrained such that we set our
>> instruments correctly on all experiments *EXCEPT* when the experimenters
>> try to test for realism or locality, then we "choose" to set them
>> incorrectly. That's why I don't understand how anyone can take this
>> seriously. That is why I think superdeterminism is bullshit.
>>
>
> Bell seemed to think that super determinism meant that the mind of the
> experimenters was determined along with everything else, which he described
> as a lack of “free will” (it seems he meant by this lack of randomness in
> their minds), which he thought was an assumption in the experiment:
>
> “There is a way to escape the inference of superluminal
>  speeds and spooky action at
> a distance. But it involves absolute determinism
>  in the universe, the complete
> absence of free will . Suppose
> the world is super-deterministic, with not just inanimate nature running on
> behind-the-scenes clockwork, but with our behavior, including our belief
> that we are free to choose to do one experiment rather than another,
> absolutely predetermined, including the ‘decision’ by the experimenter to
> carry out one set of measurements rather than another, the difficulty
> disappears. There is no need for a faster-than-light signal to tell particle
>  *A* what measurement has been carried out on particle *B*, because the
> universe, including particle *A*, already ‘knows’ what that measurement,
> and its outcome, will be.”
>
>
> --
> You received this message because you are subscribed to the Google Groups
> "Everything List" group.
> To unsubscribe from this group and stop receiving emails from it, send an
> email to everything-list+unsubscr...@googlegroups.com.
> To view this discussion on the web visit
> https://groups.google.com/d/msgid/everything-list/CAH%3D2ypWLBgv4E2ogPcOTywcg-ndqPbzxLJ3L6%2Bx62c5gGjkNBw%40mail.gmail.com
> 
> .
>

-- 
You received this message because you are subscribed to the Google Groups 
"Everything List" group.
To unsubscribe from this group and stop receiving emails from it, send an email 
to everything-list+unsubscr...@googlegroup

Re: Is Many Worlds Falsifiable?

2023-09-01 Thread Stathis Papaioannou
On Sat, 2 Sep 2023 at 04:20, John Clark  wrote:

> On Fri, Sep 1, 2023 at 1:22 PM Stathis Papaioannou 
> wrote:
>
>  >> according to superdeterminism the particular initial condition the
>>> universe was in 13.8 billion years ago has determined if you think
>>> superdeterminism is a reasonable theory or if you think it's complete
>>> bullshit. As for me I was determined to believe it's bullshit.
>>>
>>
>> *>I still struggle to see the difference between determinism and
>> superdeterminism. They both say that there is no true randomness*
>>
>
> Yes.
>
>
>> * > which includes randomness in how the experimenters set up their
>> experiment.*
>>
>
> No. Knowing the laws of physics is not enough, to make predictions you
> also need to know the initial conditions. Superdeterminism says more than a
> given state of the universe is the mathematical product of the previous
> state, superdeterminism assumes, for no particular reason, that out of the
> infinite number of states the universe could've started out at, 13.8
> billion years ago it was in the one and only one particular state that
> would make experimenters 13.8 billion years later "choose" to set their
> instruments in such a way that they always *INCORRECTLY* conclude that
> things can *NOT* be both realistic and local. It would be absolutely
> impossible to make a larger assumption than this, and that is why it is the
> largest violation of Occam's Razor conceivable. There are an infinite
> number of initial conditions the universe could've started out in and in
> which things would be deterministic today, but one and only one initial
> condition would produce the universe in which superdeterminism is true. And
> if superdeterminism were true then there would be no point in performing
> scientific experiments since there would be no reason for them to lead to
> the truth, and yet airplanes fly and bridges don't collapse so they do seem
> to lead to the truth, there is no way to explain that unless the initial
> conditions were even further restrained such that we set our instruments
> correctly on all experiments *EXCEPT* when the experimenters try to test
> for realism or locality, then we "choose" to set them incorrectly. That's
> why I don't understand how anyone can take this seriously. That is why I
> think superdeterminism is bullshit.
>

Bell seemed to think that super determinism meant that the mind of the
experimenters was determined along with everything else, which he described
as a lack of “free will” (it seems he meant by this lack of randomness in
their minds), which he thought was an assumption in the experiment:

“There is a way to escape the inference of superluminal
 speeds and spooky action at a
distance. But it involves absolute determinism
 in the universe, the complete
absence of free will . Suppose the
world is super-deterministic, with not just inanimate nature running on
behind-the-scenes clockwork, but with our behavior, including our belief
that we are free to choose to do one experiment rather than another,
absolutely predetermined, including the ‘decision’ by the experimenter to
carry out one set of measurements rather than another, the difficulty
disappears. There is no need for a faster-than-light signal to tell particle
 *A* what measurement has been carried out on particle *B*, because the
universe, including particle *A*, already ‘knows’ what that measurement,
and its outcome, will be.”

-- 
You received this message because you are subscribed to the Google Groups 
"Everything List" group.
To unsubscribe from this group and stop receiving emails from it, send an email 
to everything-list+unsubscr...@googlegroups.com.
To view this discussion on the web visit 
https://groups.google.com/d/msgid/everything-list/CAH%3D2ypWLBgv4E2ogPcOTywcg-ndqPbzxLJ3L6%2Bx62c5gGjkNBw%40mail.gmail.com.


Re: Is Many Worlds Falsifiable?

2023-09-01 Thread John Clark
On Fri, Sep 1, 2023 at 1:22 PM Stathis Papaioannou 
wrote:

 >> according to superdeterminism the particular initial condition the
>> universe was in 13.8 billion years ago has determined if you think
>> superdeterminism is a reasonable theory or if you think it's complete
>> bullshit. As for me I was determined to believe it's bullshit.
>>
>
> *>I still struggle to see the difference between determinism and
> superdeterminism. They both say that there is no true randomness*
>

Yes.


> * > which includes randomness in how the experimenters set up their
> experiment.*
>

No. Knowing the laws of physics is not enough, to make predictions you also
need to know the initial conditions. Superdeterminism says more than a
given state of the universe is the mathematical product of the previous
state, superdeterminism assumes, for no particular reason, that out of the
infinite number of states the universe could've started out at, 13.8
billion years ago it was in the one and only one particular state that
would make experimenters 13.8 billion years later "choose" to set their
instruments in such a way that they always *INCORRECTLY* conclude that
things can *NOT* be both realistic and local. It would be absolutely
impossible to make a larger assumption than this, and that is why it is the
largest violation of Occam's Razor conceivable. There are an infinite
number of initial conditions the universe could've started out in and in
which things would be deterministic today, but one and only one initial
condition would produce the universe in which superdeterminism is true. And
if superdeterminism were true then there would be no point in performing
scientific experiments since there would be no reason for them to lead to
the truth, and yet airplanes fly and bridges don't collapse so they do seem
to lead to the truth, there is no way to explain that unless the initial
conditions were even further restrained such that we set our instruments
correctly on all experiments *EXCEPT* when the experimenters try to test
for realism or locality, then we "choose" to set them incorrectly. That's
why I don't understand how anyone can take this seriously. That is why I
think superdeterminism is bullshit.

 John K ClarkSee what's on my new list at  Extropolis

dss

-- 
You received this message because you are subscribed to the Google Groups 
"Everything List" group.
To unsubscribe from this group and stop receiving emails from it, send an email 
to everything-list+unsubscr...@googlegroups.com.
To view this discussion on the web visit 
https://groups.google.com/d/msgid/everything-list/CAJPayv0MqLP%3DDFXZ3pmUsuPUAw%2BfCn-0XVV8xfLLOQsk5%2BrPmA%40mail.gmail.com.


Re: Is Many Worlds Falsifiable?

2023-09-01 Thread Stathis Papaioannou
On Sat, 2 Sep 2023 at 00:03, John Clark  wrote:

> On Fri, Sep 1, 2023 at 9:54 AM Jason Resch  wrote:
>
> *> But did (or could) superdeterminism choose the digits of Pi?*
>
>
> According to superdeterminism, yes. And according to superdeterminism the
> particular initial condition the universe was in 13.8 billion years ago has
> determined if you think superdeterminism is a reasonable theory or if you
> think it's complete bullshit. As for me I was determined to believe it's
> bullshit.
>

I still struggle to see the difference between determinism and
superdeterminism. They both say that there is no true randomness, which
includes randomness in how the experimenters set up their experiment. If
everything is determined, then everything is perfectly correlated from the
start of time. If you think it’s odd that it should be correlated in a
particular way, that’s your problem, because it’s the universe we live in.

 John K ClarkSee what's on my new list at  Extropolis
> 
> iqf
>
>
>
> --
> You received this message because you are subscribed to the Google Groups
> "Everything List" group.
> To unsubscribe from this group and stop receiving emails from it, send an
> email to everything-list+unsubscr...@googlegroups.com.
> To view this discussion on the web visit
> https://groups.google.com/d/msgid/everything-list/CAJPayv18bQUH8%3DmdXkhwZ%2Bbg6Gvb8rr6mtVRd0MUxxEGiqqOaw%40mail.gmail.com
> 
> .
>

-- 
You received this message because you are subscribed to the Google Groups 
"Everything List" group.
To unsubscribe from this group and stop receiving emails from it, send an email 
to everything-list+unsubscr...@googlegroups.com.
To view this discussion on the web visit 
https://groups.google.com/d/msgid/everything-list/CAH%3D2ypVgRV8AVVfn%2BonuuPLGxTuNBs0fTcVp0j%2BcNuTxXDaMTw%40mail.gmail.com.


Re: Is Many Worlds Falsifiable?

2023-09-01 Thread John Clark
On Fri, Sep 1, 2023 at 9:54 AM Jason Resch  wrote:

*> But did (or could) superdeterminism choose the digits of Pi?*


According to superdeterminism, yes. And according to superdeterminism the
particular initial condition the universe was in 13.8 billion years ago has
determined if you think superdeterminism is a reasonable theory or if you
think it's complete bullshit. As for me I was determined to believe it's
bullshit.

 John K ClarkSee what's on my new list at  Extropolis

iqf

-- 
You received this message because you are subscribed to the Google Groups 
"Everything List" group.
To unsubscribe from this group and stop receiving emails from it, send an email 
to everything-list+unsubscr...@googlegroups.com.
To view this discussion on the web visit 
https://groups.google.com/d/msgid/everything-list/CAJPayv18bQUH8%3DmdXkhwZ%2Bbg6Gvb8rr6mtVRd0MUxxEGiqqOaw%40mail.gmail.com.


Re: Is Many Worlds Falsifiable?

2023-09-01 Thread Jason Resch
On Fri, Sep 1, 2023 at 8:52 AM John Clark  wrote:

>
>
> On Fri, Sep 1, 2023 at 9:38 AM Jason Resch  wrote:
>
>
>
>> >> 128 bits would probably be enough information to program a Turing
>>> Machine to calculate the infinite series 4(1-1/3 +1/5 -1/7 +...) and
>>> that would produce an infinite string of digits that never repeats and
>>> looks completely random, 31415926535
>>> 897932384626433832795028841971693993751058209749445923078164062862089986280348253421170679
>>> ., because that particular infinite series converges to the
>>> transcendental number *π*.
>>>
>>
>> *> It's not that it's generating apparent random results though,
>> superdeterminism requires results that are correlated to the way we choose
>> to make the measurements.*
>>
>
> But according to superdeterminism your "choices" of how to make the
> measurements were also completely determined, if you had "chosen" to make
> the measurements in a certain way you could have shown that
> superdeterminism produce results that were self-contradictory, but you have
> never "chosen" to do so and you never will.  By the way, I feel a little
> queasy defending superdeterminism because I think the idea is completely
> idiotic.
>

But did (or could) superdeterminism choose the digits of Pi?

Jason



>
>   John K ClarkSee what's on my new list at  Extropolis
> 
> ifq
>
>
>
 On Fri, Sep 1, 2023 at 7:26 AM John Clark  wrote:

> On Thu, Aug 31, 2023 at 6:29 PM Bruce Kellett 
> wrote:
>
> *> OK. So spell out your non-realist, but local, many worlds account
>> of the violations of the Bell inequalities. It seems that you want it 
>> both
>> ways -- Bell's theorem says that MWI must be non-local, but you claim 
>> that
>> it is local? "Realism" has nothing to do with it.*
>
>
>
> "Realism" has* EVERYTHING* to do with it, and I spelled out exactly
> why in a post on May 4 2022 when somebody said they wanted to hear all the
> gory details and this is what I said:
> ==
>
> " If you want all the details this is going to be a long post, you
> asked for it. First I'm gonna have to show that any theory (except for
> superdeterminism which is idiotic) that is deterministic, local and
> realistic cannot possibly explain the violation of Bell's Inequality that
> we see in our experiments, and then show why *a theory like Many
> Worlds which is deterministic and local but NOT realistic can.*
>
> The hidden variable concept was Einstein's idea, he thought there was
> a local reason all events happened, even quantum mechanical events,
> but we just can't see what they are. It was a reasonable guess at the time
> but today experiments have shown that Einstein was wrong, to do that I'm
> gonna illustrate some of the details of Bell's inequality with an example.
>
> When a photon of undetermined polarization hits a polarizing filter
> there is a 50% chance it will make it through. For many years physicists
> like Einstein who disliked the idea that God played dice with the universe
> figured there must be a hidden variable inside the photon that told it 
> what
> to do. By "hidden variable" they meant something different about that
> particular photon that we just don't know about. They meant something
> equivalent to a look-up table inside the photon that for one reason or
> another we are unable to access but the photon can when it wants to know 
> if
> it should go through a filter or be stopped by one. We now understand that
> is impossible. In 1964 (but not published until 1967) John Bell showed 
> that
> correlations that work by hidden variables must be less than or equal to a
> certain value, this is called Bell's inequality. In experiment it was 
> found
> that some correlations are actually greater than that value. Quantum
> Mechanics can explain this, classical physics or even classical logic can
> not.
>
> Even if Quantum Mechanics is someday proven to be untrue Bell's
> argument is still valid, in fact his original paper had no Quantum
> Mechanics in it and can be derived with high school algebra; his point was
> that any successful theory about how the world works must explain why his
> inequality is violated, and today we know for a fact from experiments
> that it is indeed violated. Nature just refuses to be sensible and doesn't
> work the way you'd think it should.
>
> I have a black box, it has a red light and a blue light on it, it also
> has a rotary switch with 6 connections at the 12,2,4,6,8 and 10 o'clock
> positions. The red and blue light blink in a manner that passes all known
> tests for being completely random, this is true regardless of what 
> position
> the rotary switch is in. Such a box could be made and still be completely
> determ

Re: Is Many Worlds Falsifiable?

2023-09-01 Thread John Clark
On Fri, Sep 1, 2023 at 9:38 AM Jason Resch  wrote:



> >> 128 bits would probably be enough information to program a Turing
>> Machine to calculate the infinite series 4(1-1/3 +1/5 -1/7 +...) and
>> that would produce an infinite string of digits that never repeats and
>> looks completely random, 31415926535
>> 897932384626433832795028841971693993751058209749445923078164062862089986280348253421170679
>> ., because that particular infinite series converges to the
>> transcendental number *π*.
>>
>
> *> It's not that it's generating apparent random results though,
> superdeterminism requires results that are correlated to the way we choose
> to make the measurements.*
>

But according to superdeterminism your "choices" of how to make the
measurements were also completely determined, if you had "chosen" to make
the measurements in a certain way you could have shown that
superdeterminism produce results that were self-contradictory, but you have
never "chosen" to do so and you never will.  By the way, I feel a little
queasy defending superdeterminism because I think the idea is completely
idiotic.

  John K ClarkSee what's on my new list at  Extropolis

ifq



>>> On Fri, Sep 1, 2023 at 7:26 AM John Clark  wrote:
>>>
 On Thu, Aug 31, 2023 at 6:29 PM Bruce Kellett 
 wrote:

 *> OK. So spell out your non-realist, but local, many worlds account of
> the violations of the Bell inequalities. It seems that you want it both
> ways -- Bell's theorem says that MWI must be non-local, but you claim that
> it is local? "Realism" has nothing to do with it.*



 "Realism" has* EVERYTHING* to do with it, and I spelled out exactly
 why in a post on May 4 2022 when somebody said they wanted to hear all the
 gory details and this is what I said:
 ==

 " If you want all the details this is going to be a long post, you
 asked for it. First I'm gonna have to show that any theory (except for
 superdeterminism which is idiotic) that is deterministic, local and
 realistic cannot possibly explain the violation of Bell's Inequality that
 we see in our experiments, and then show why *a theory like Many
 Worlds which is deterministic and local but NOT realistic can.*

 The hidden variable concept was Einstein's idea, he thought there was a
 local reason all events happened, even quantum mechanical events, but
 we just can't see what they are. It was a reasonable guess at the time but
 today experiments have shown that Einstein was wrong, to do that I'm gonna
 illustrate some of the details of Bell's inequality with an example.

 When a photon of undetermined polarization hits a polarizing filter
 there is a 50% chance it will make it through. For many years physicists
 like Einstein who disliked the idea that God played dice with the universe
 figured there must be a hidden variable inside the photon that told it what
 to do. By "hidden variable" they meant something different about that
 particular photon that we just don't know about. They meant something
 equivalent to a look-up table inside the photon that for one reason or
 another we are unable to access but the photon can when it wants to know if
 it should go through a filter or be stopped by one. We now understand that
 is impossible. In 1964 (but not published until 1967) John Bell showed that
 correlations that work by hidden variables must be less than or equal to a
 certain value, this is called Bell's inequality. In experiment it was found
 that some correlations are actually greater than that value. Quantum
 Mechanics can explain this, classical physics or even classical logic can
 not.

 Even if Quantum Mechanics is someday proven to be untrue Bell's
 argument is still valid, in fact his original paper had no Quantum
 Mechanics in it and can be derived with high school algebra; his point was
 that any successful theory about how the world works must explain why his
 inequality is violated, and today we know for a fact from experiments
 that it is indeed violated. Nature just refuses to be sensible and doesn't
 work the way you'd think it should.

 I have a black box, it has a red light and a blue light on it, it also
 has a rotary switch with 6 connections at the 12,2,4,6,8 and 10 o'clock
 positions. The red and blue light blink in a manner that passes all known
 tests for being completely random, this is true regardless of what position
 the rotary switch is in. Such a box could be made and still be completely
 deterministic by just pre-computing 6 different random sequences and
 recording them as a look-up table in the box. Now the box would know which
 light to flash.

 I have another black box. When both boxes have the same setting on
 their rotary switch they both produce

Re: Is Many Worlds Falsifiable?

2023-09-01 Thread Jason Resch
On Fri, Sep 1, 2023, 9:16 AM John Clark  wrote:

> On Fri, Sep 1, 2023 at 8:41 AM Jason Resch  wrote:
>
> *> I think it may be possible actually, to use a mathematical argument to
>> disprove superdeterminism*
>>
>
> I'm not sure a mathematical proof that superdeterminism is not true is
> even necessary because a greater violation of Occam's Razor is quite
> literally impossible to imagine.
>
> *> it's not feasible for 128 measurements, to mathematically, contain
>> enough information and variation to also determine and the subsequent 2^128
>> outcomes.*
>
>
> 128 bits would probably be enough information to program a Turing Machine
> to calculate the infinite series 4(1-1/3 +1/5 -1/7 +...) and that would
> produce an infinite string of digits that never repeats and looks
> completely random, 31415926535
> 897932384626433832795028841971693993751058209749445923078164062862089986280348253421170679
> ., because that particular infinite series converges to the
> transcendental number *π*.
>

It's not that it's generating apparent random results though,
superdeterminism requires results that are correlated to the way we choose
to make the measurements.

So how can these correlations be predetermined to follow the outputs of
this algorithm, when the deterministic algorithm is deciding what
measurements to make? And the deterministic algorithm in question was
chosen (deterministically) from prior measurements.

It has the feeling to me of a compression algorithm that could make any
input smaller, but still perfectly decompress and return the original
input. This is impossible because there are more larger messages than
smaller ones, so the original input would be under determined.

With super determinism, every successive state of the universe is perfectly
one-to-one. But this seems like it must down whenever we try to link the
superdeterminism measurements results to against other functions that have
many, or an infinite number of, outputs from one input or initial state.

Jason



>
>  John K ClarkSee what's on my new list at  Extropolis
> 
>
> isc
>
>
>> On Fri, Sep 1, 2023 at 7:26 AM John Clark  wrote:
>>
>>> On Thu, Aug 31, 2023 at 6:29 PM Bruce Kellett 
>>> wrote:
>>>
>>> *> OK. So spell out your non-realist, but local, many worlds account of
 the violations of the Bell inequalities. It seems that you want it both
 ways -- Bell's theorem says that MWI must be non-local, but you claim that
 it is local? "Realism" has nothing to do with it.*
>>>
>>>
>>>
>>> "Realism" has* EVERYTHING* to do with it, and I spelled out exactly why
>>> in a post on May 4 2022 when somebody said they wanted to hear all the gory
>>> details and this is what I said:
>>> ==
>>>
>>> " If you want all the details this is going to be a long post, you asked
>>> for it. First I'm gonna have to show that any theory (except for
>>> superdeterminism which is idiotic) that is deterministic, local and
>>> realistic cannot possibly explain the violation of Bell's Inequality that
>>> we see in our experiments, and then show why *a theory like Many Worlds
>>> which is deterministic and local but NOT realistic can.*
>>>
>>> The hidden variable concept was Einstein's idea, he thought there was a
>>> local reason all events happened, even quantum mechanical events, but
>>> we just can't see what they are. It was a reasonable guess at the time but
>>> today experiments have shown that Einstein was wrong, to do that I'm gonna
>>> illustrate some of the details of Bell's inequality with an example.
>>>
>>> When a photon of undetermined polarization hits a polarizing filter
>>> there is a 50% chance it will make it through. For many years physicists
>>> like Einstein who disliked the idea that God played dice with the universe
>>> figured there must be a hidden variable inside the photon that told it what
>>> to do. By "hidden variable" they meant something different about that
>>> particular photon that we just don't know about. They meant something
>>> equivalent to a look-up table inside the photon that for one reason or
>>> another we are unable to access but the photon can when it wants to know if
>>> it should go through a filter or be stopped by one. We now understand that
>>> is impossible. In 1964 (but not published until 1967) John Bell showed that
>>> correlations that work by hidden variables must be less than or equal to a
>>> certain value, this is called Bell's inequality. In experiment it was found
>>> that some correlations are actually greater than that value. Quantum
>>> Mechanics can explain this, classical physics or even classical logic can
>>> not.
>>>
>>> Even if Quantum Mechanics is someday proven to be untrue Bell's
>>> argument is still valid, in fact his original paper had no Quantum
>>> Mechanics in it and can be derived with high school algebra; his point was
>>> that any successful theory about how the world works must explain why his
>>> inequality is violated, a

Re: Is Many Worlds Falsifiable?

2023-09-01 Thread John Clark
On Fri, Sep 1, 2023 at 8:41 AM Jason Resch  wrote:

*> I think it may be possible actually, to use a mathematical argument to
> disprove superdeterminism*
>

I'm not sure a mathematical proof that superdeterminism is not true is even
necessary because a greater violation of Occam's Razor is quite literally
impossible to imagine.

*> it's not feasible for 128 measurements, to mathematically, contain
> enough information and variation to also determine and the subsequent 2^128
> outcomes.*


128 bits would probably be enough information to program a Turing Machine
to calculate the infinite series 4(1-1/3 +1/5 -1/7 +...) and that would
produce an infinite string of digits that never repeats and looks
completely random, 31415926535
897932384626433832795028841971693993751058209749445923078164062862089986280348253421170679
., because that particular infinite series converges to the
transcendental number *π*.

 John K ClarkSee what's on my new list at  Extropolis


isc


> On Fri, Sep 1, 2023 at 7:26 AM John Clark  wrote:
>
>> On Thu, Aug 31, 2023 at 6:29 PM Bruce Kellett 
>> wrote:
>>
>> *> OK. So spell out your non-realist, but local, many worlds account of
>>> the violations of the Bell inequalities. It seems that you want it both
>>> ways -- Bell's theorem says that MWI must be non-local, but you claim that
>>> it is local? "Realism" has nothing to do with it.*
>>
>>
>>
>> "Realism" has* EVERYTHING* to do with it, and I spelled out exactly why
>> in a post on May 4 2022 when somebody said they wanted to hear all the gory
>> details and this is what I said:
>> ==
>>
>> " If you want all the details this is going to be a long post, you asked
>> for it. First I'm gonna have to show that any theory (except for
>> superdeterminism which is idiotic) that is deterministic, local and
>> realistic cannot possibly explain the violation of Bell's Inequality that
>> we see in our experiments, and then show why *a theory like Many Worlds
>> which is deterministic and local but NOT realistic can.*
>>
>> The hidden variable concept was Einstein's idea, he thought there was a
>> local reason all events happened, even quantum mechanical events, but we
>> just can't see what they are. It was a reasonable guess at the time but
>> today experiments have shown that Einstein was wrong, to do that I'm gonna
>> illustrate some of the details of Bell's inequality with an example.
>>
>> When a photon of undetermined polarization hits a polarizing filter there
>> is a 50% chance it will make it through. For many years physicists like
>> Einstein who disliked the idea that God played dice with the universe
>> figured there must be a hidden variable inside the photon that told it what
>> to do. By "hidden variable" they meant something different about that
>> particular photon that we just don't know about. They meant something
>> equivalent to a look-up table inside the photon that for one reason or
>> another we are unable to access but the photon can when it wants to know if
>> it should go through a filter or be stopped by one. We now understand that
>> is impossible. In 1964 (but not published until 1967) John Bell showed that
>> correlations that work by hidden variables must be less than or equal to a
>> certain value, this is called Bell's inequality. In experiment it was found
>> that some correlations are actually greater than that value. Quantum
>> Mechanics can explain this, classical physics or even classical logic can
>> not.
>>
>> Even if Quantum Mechanics is someday proven to be untrue Bell's argument
>> is still valid, in fact his original paper had no Quantum Mechanics in it
>> and can be derived with high school algebra; his point was that any
>> successful theory about how the world works must explain why his
>> inequality is violated, and today we know for a fact from experiments
>> that it is indeed violated. Nature just refuses to be sensible and doesn't
>> work the way you'd think it should.
>>
>> I have a black box, it has a red light and a blue light on it, it also
>> has a rotary switch with 6 connections at the 12,2,4,6,8 and 10 o'clock
>> positions. The red and blue light blink in a manner that passes all known
>> tests for being completely random, this is true regardless of what position
>> the rotary switch is in. Such a box could be made and still be completely
>> deterministic by just pre-computing 6 different random sequences and
>> recording them as a look-up table in the box. Now the box would know which
>> light to flash.
>>
>> I have another black box. When both boxes have the same setting on their
>> rotary switch they both produce the same random sequence of light flashes.
>> This would also be easy to reproduce in a classical physics world, just
>> record the same 6 random sequences in both boxes.
>>
>> The set of boxes has another property, if the switches on the 2 boxes are
>> set to opposite positions, 12 and 6 o'clock for example, there is a tota

Re: Is Many Worlds Falsifiable?

2023-09-01 Thread Jason Resch
I think it may be possible actually, to use a mathematical argument to
disprove superdeterminism, in a manner similar to how Bell disproved
theories that are local, real, and counterfactually definite.

The method would show that there is a necessary underdetermination that can
happen, when a small number of measurement results are gathered, and then
used to feed back into the polarizing settings for a much larger number of
subsequent measurements. If the universe is completely deterministic, as
superdeterminism proposes, there should be a point at which the
correlations must fail, as there are not enough ways a single (or small
number of) facts can determine a much larger, potentially infinite, number
of following facts.

As an example, take the first 128 measurements from a Bell type experiment,
and use the measured values to determine the 128-bits of an encryption key.
Use that key to initialize a cipher (which can be viewed as a seed to a
pseudo random number generator), which has a period of 2^128. That is, this
cipher (or pseudo random number generator), will output a deterministic
sequence of bits that is on the order of 2^128 bits long. Use these output
bits to determine the settings of how to set the angle of the polarizing
filter in an iterated Bell/EPR test.

According to Superdeterminism, everything measured was pre-determined to
have the measurement results. However, in this case, it's not feasible for
128 measurements, to mathematically, contain enough information and
variation to also determine and the subsequent 2^128 outcomes. The 2^128
outcomes are mathematically underdetermined by 128 prior measurements, and
so the system cannot be deterministic in the way superdeterminism proposes.

Jason


On Fri, Sep 1, 2023 at 7:26 AM John Clark  wrote:

> On Thu, Aug 31, 2023 at 6:29 PM Bruce Kellett 
> wrote:
>
> *> OK. So spell out your non-realist, but local, many worlds account of
>> the violations of the Bell inequalities. It seems that you want it both
>> ways -- Bell's theorem says that MWI must be non-local, but you claim that
>> it is local? "Realism" has nothing to do with it.*
>
>
>
> "Realism" has* EVERYTHING* to do with it, and I spelled out exactly why
> in a post on May 4 2022 when somebody said they wanted to hear all the gory
> details and this is what I said:
> ==
>
> " If you want all the details this is going to be a long post, you asked
> for it. First I'm gonna have to show that any theory (except for
> superdeterminism which is idiotic) that is deterministic, local and
> realistic cannot possibly explain the violation of Bell's Inequality that
> we see in our experiments, and then show why *a theory like Many Worlds
> which is deterministic and local but NOT realistic can.*
>
> The hidden variable concept was Einstein's idea, he thought there was a
> local reason all events happened, even quantum mechanical events, but we
> just can't see what they are. It was a reasonable guess at the time but
> today experiments have shown that Einstein was wrong, to do that I'm gonna
> illustrate some of the details of Bell's inequality with an example.
>
> When a photon of undetermined polarization hits a polarizing filter there
> is a 50% chance it will make it through. For many years physicists like
> Einstein who disliked the idea that God played dice with the universe
> figured there must be a hidden variable inside the photon that told it what
> to do. By "hidden variable" they meant something different about that
> particular photon that we just don't know about. They meant something
> equivalent to a look-up table inside the photon that for one reason or
> another we are unable to access but the photon can when it wants to know if
> it should go through a filter or be stopped by one. We now understand that
> is impossible. In 1964 (but not published until 1967) John Bell showed that
> correlations that work by hidden variables must be less than or equal to a
> certain value, this is called Bell's inequality. In experiment it was found
> that some correlations are actually greater than that value. Quantum
> Mechanics can explain this, classical physics or even classical logic can
> not.
>
> Even if Quantum Mechanics is someday proven to be untrue Bell's argument
> is still valid, in fact his original paper had no Quantum Mechanics in it
> and can be derived with high school algebra; his point was that any
> successful theory about how the world works must explain why his
> inequality is violated, and today we know for a fact from experiments
> that it is indeed violated. Nature just refuses to be sensible and doesn't
> work the way you'd think it should.
>
> I have a black box, it has a red light and a blue light on it, it also has
> a rotary switch with 6 connections at the 12,2,4,6,8 and 10 o'clock
> positions. The red and blue light blink in a manner that passes all known
> tests for being completely random, this is true regardless of what position
> the rotary switch is in. Such 

Re: Is Many Worlds Falsifiable?

2023-09-01 Thread John Clark
On Thu, Aug 31, 2023 at 6:29 PM Bruce Kellett  wrote:

*> OK. So spell out your non-realist, but local, many worlds account of the
> violations of the Bell inequalities. It seems that you want it both ways --
> Bell's theorem says that MWI must be non-local, but you claim that it is
> local? "Realism" has nothing to do with it.*



"Realism" has* EVERYTHING* to do with it, and I spelled out exactly why in
a post on May 4 2022 when somebody said they wanted to hear all the gory
details and this is what I said:
==

" If you want all the details this is going to be a long post, you asked
for it. First I'm gonna have to show that any theory (except for
superdeterminism which is idiotic) that is deterministic, local and
realistic cannot possibly explain the violation of Bell's Inequality that
we see in our experiments, and then show why *a theory like Many Worlds
which is deterministic and local but NOT realistic can.*

The hidden variable concept was Einstein's idea, he thought there was a
local reason all events happened, even quantum mechanical events, but we
just can't see what they are. It was a reasonable guess at the time but
today experiments have shown that Einstein was wrong, to do that I'm gonna
illustrate some of the details of Bell's inequality with an example.

When a photon of undetermined polarization hits a polarizing filter there
is a 50% chance it will make it through. For many years physicists like
Einstein who disliked the idea that God played dice with the universe
figured there must be a hidden variable inside the photon that told it what
to do. By "hidden variable" they meant something different about that
particular photon that we just don't know about. They meant something
equivalent to a look-up table inside the photon that for one reason or
another we are unable to access but the photon can when it wants to know if
it should go through a filter or be stopped by one. We now understand that
is impossible. In 1964 (but not published until 1967) John Bell showed that
correlations that work by hidden variables must be less than or equal to a
certain value, this is called Bell's inequality. In experiment it was found
that some correlations are actually greater than that value. Quantum
Mechanics can explain this, classical physics or even classical logic can
not.

Even if Quantum Mechanics is someday proven to be untrue Bell's argument is
still valid, in fact his original paper had no Quantum Mechanics in it and
can be derived with high school algebra; his point was that any successful
theory about how the world works must explain why his inequality is
violated, and today we know for a fact from experiments that it is indeed
violated. Nature just refuses to be sensible and doesn't work the way you'd
think it should.

I have a black box, it has a red light and a blue light on it, it also has
a rotary switch with 6 connections at the 12,2,4,6,8 and 10 o'clock
positions. The red and blue light blink in a manner that passes all known
tests for being completely random, this is true regardless of what position
the rotary switch is in. Such a box could be made and still be completely
deterministic by just pre-computing 6 different random sequences and
recording them as a look-up table in the box. Now the box would know which
light to flash.

I have another black box. When both boxes have the same setting on their
rotary switch they both produce the same random sequence of light flashes.
This would also be easy to reproduce in a classical physics world, just
record the same 6 random sequences in both boxes.

The set of boxes has another property, if the switches on the 2 boxes are
set to opposite positions, 12 and 6 o'clock for example, there is a total
negative correlation; when one flashes red the other box flashes blue and
when one box flashes blue the other flashes red. This just makes it all the
easier to make the boxes because now you only need to pre-calculate 3
random sequences, then just change every 1 to 0 and every 0 to 1 to get the
other 3 sequences and record all 6 in both boxes.

The boxes have one more feature that makes things very interesting, if the
rotary switch on a box is one notch different from the setting on the other
box then the sequence of light flashes will on average be different 1 time
in 4. How on Earth could I make the boxes behave like that? Well, I could
change on average one entry in 4 of the 12 o'clock look-up table (hidden
variable) sequence and make that the 2 o'clock table. Then change 1 in 4 of
the 2 o'clock and make that the 4 o'clock, and change 1 in 4 of the 4
o'clock and make that the 6 o'clock. So now the light flashes on the box
set at 2 o'clock is different from the box set at 12 o'clock on average by
1 flash in 4. The box set at 4 o'clock differs from the one set at 12 by 2
flashes in 4, and the one set at 6 differs from the one set at 12 by 3
flashes in 4.

BUT I said before that boxes with opposite settings should have a 100%
anti-correlation, the f

Re: Is Many Worlds Falsifiable?

2023-08-31 Thread Bruce Kellett
On Thu, Aug 31, 2023 at 10:07 PM John Clark  wrote:

> On Thu, Aug 31, 2023 at 7:24 AM Bruce Kellett 
> wrote:
>
> *>> Well of course it isn't! Bell's Inequality has been experimentally
>>> shown to be violated, so if there are hidden variables they can't be local.
>>> *
>>>
>>
>> *> But the argument was that many worlds was an entirely local theory: in
>> other words, that it gives a local account of the violation of the Bell
>> inequalities.*
>>
>
> Well that isn't my argument!
>
>
>> *> it seems from what you say that you agree that Bell's theorem proves
>> that no local account of the experimental results for correlations of
>> entangled particles is possible. I agree.*
>>
>
>  Bell's theorem proves that no REALISTIC local account can explain the
> experimental fact that Bell's Inequality is violated.
>
>
>> * > But that is not what is claimed by Saibal and other advocates of MWI:*
>>
>
> The violation of Bell's Inequality proves that no theory that is both
> realistic and local can be right. I think Many Worlds is local because you
> cannot send information faster than light in that theory, apparently you
> disagree and for some reason think Many Worlds is non-local, but as far
> as this discussion is concerned it doesn't matter which of us is right
> because Many Worlds is *NOT* a realistic theory. "Realistic" means that
> unobserved things exist in one and only one definite state, and that is
> most certainly not what Many Worlds says.
>

OK. So spell out your non-realist, but local, many worlds account of the
violations of the Bell inequalities. It seems that you want it both ways --
Bell's theorem says that MWI must be non-local, but you claim that it is
local? "Realism" has nothing to do with it.

Bruce

-- 
You received this message because you are subscribed to the Google Groups 
"Everything List" group.
To unsubscribe from this group and stop receiving emails from it, send an email 
to everything-list+unsubscr...@googlegroups.com.
To view this discussion on the web visit 
https://groups.google.com/d/msgid/everything-list/CAFxXSLQ6h5rS27_t1jYe9MKEupc6qSbWj3nF9SGKn02Ogb%3DWtA%40mail.gmail.com.


Re: Is Many Worlds Falsifiable?

2023-08-31 Thread John Clark
On Thu, Aug 31, 2023 at 7:24 AM Bruce Kellett  wrote:

*>> Well of course it isn't! Bell's Inequality has been experimentally
>> shown to be violated, so if there are hidden variables they can't be local.
>> *
>>
>
> *> But the argument was that many worlds was an entirely local theory: in
> other words, that it gives a local account of the violation of the Bell
> inequalities.*
>

Well that isn't my argument!


> *> it seems from what you say that you agree that Bell's theorem proves
> that no local account of the experimental results for correlations of
> entangled particles is possible. I agree.*
>

 Bell's theorem proves that no REALISTIC local account can explain the
experimental fact that Bell's Inequality is violated.


> * > But that is not what is claimed by Saibal and other advocates of MWI:*
>

The violation of Bell's Inequality proves that no theory that is both
realistic and local can be right. I think Many Worlds is local because you
cannot send information faster than light in that theory, apparently you
disagree and for some reason think Many Worlds is non-local, but as far as
this discussion is concerned it doesn't matter which of us is right because
Many Worlds is *NOT* a realistic theory. "Realistic" means that unobserved
things exist in one and only one definite state, and that is most certainly
not what Many Worlds says.

John K ClarkSee what's on my new list at  Extropolis

rmu

iyd


>

-- 
You received this message because you are subscribed to the Google Groups 
"Everything List" group.
To unsubscribe from this group and stop receiving emails from it, send an email 
to everything-list+unsubscr...@googlegroups.com.
To view this discussion on the web visit 
https://groups.google.com/d/msgid/everything-list/CAJPayv1w2PsuEFHe71C_oSR8Q9aZpimTx0mOfjb1AsrKxYdUWA%40mail.gmail.com.


Re: Is Many Worlds Falsifiable?

2023-08-31 Thread Bruce Kellett
On Thu, Aug 31, 2023 at 9:12 PM John Clark  wrote:

> On Thu, Aug 31, 2023 at 12:09 AM Bruce Kellett 
> wrote:
>
> *>> On Thu, Aug 31, 2023 at 12:27 PM smitra >> > wrote:*
>>
>>
>>>
>>>
>>>
>>>
>>>
>>>
>>>
>>> *There is no problem here because in practice MWI is nothing more than
>>> the usual QM formalism to compute the outcome of experiments where you then
>>> assume that the ensemble of all possible outcomes really exists. Locality
>>> then follows from the fact hat the dynamics of the theory is manifestly
>>> local. The Hamiltonian only includes local interactions and observers are
>>> part of this dynamics. Although observer are not explicitly treated as
>>> being part of the wavefunction that describes the entire system, the
>>> assumption is that in principle, this is the case. In practice, one can
>>> then proceed according to the usual QM formalism.*
>>>
>>
>> *> That is all very well, but it is not a local account of violations of
>> the Bell inequalities.*
>>
>
> *Well of course it isn't! Bell's Inequality has been experimentally shown
> to be violated, so if there are hidden variables they can't be local. *
>

But the argument was that many worlds was an entirely local theory: in
other words, that it gives a local account of the violation of the Bell
inequalities. It seems from what you say that you agree that Bell's theorem
proves that no local account of the experimental results for correlations
of entangled particles is possible. I agree. But that is not what is
claimed by Saibal and other advocates of MWI: they claim to have a local
account of these results.

Bruce

-- 
You received this message because you are subscribed to the Google Groups 
"Everything List" group.
To unsubscribe from this group and stop receiving emails from it, send an email 
to everything-list+unsubscr...@googlegroups.com.
To view this discussion on the web visit 
https://groups.google.com/d/msgid/everything-list/CAFxXSLTY%3DHo-PZ2mc-eWVqQe5BwAyFWc_C4ezZjwRmZQr6vhmw%40mail.gmail.com.


Is Many Worlds Falsifiable?

2023-08-31 Thread John Clark
On Thu, Aug 31, 2023 at 12:09 AM Bruce Kellett 
wrote:

*>> On Thu, Aug 31, 2023 at 12:27 PM smitra > > wrote:*
>
>
>>
>>
>>
>>
>>
>>
>>
>> *There is no problem here because in practice MWI is nothing more thanthe
>> usual QM formalism to compute the outcome of experiments where youthen
>> assume that the ensemble of all possible outcomes really exists.Locality
>> then follows from the fact hat the dynamics of the theory ismanifestly
>> local. The Hamiltonian only includes local interactions andobservers are
>> part of this dynamics. Although observer are notexplicitly treated as being
>> part of the wavefunction that describes theentire system, the assumption is
>> that in principle, this is the case. Inpractice, one can then proceed
>> according to the usual QM formalism.*
>>
>
> *> That is all very well, but it is not a local account of violations of
> the Bell inequalities.*
>

*Well of course it isn't! Bell's Inequality has been experimentally shown
to be violated, so if there are hidden variables they can't be local. *

> * Bell'e theorem applies in Everettian quantum mechanics in exactly the
> same way as it applies in one-world accounts.*
>

*Bell's Inequality applies to everything, even if Quantum Mechanics were
someday proved to be wrong Bell's theorem would still be valid. Bell didn't
need Quantum Mechanics to derive his inequality, he just needed logic and
high school algebra.  So even if  someday something supplanted Quantum
Mechanics that new theory would still have to conform to the fact
that Bell's Inequality is violated.*


> *>Bell's theorem proves that the effect is non-local, so no local account
> is possible in any interpretation of QM.*
>

*Exactly. And "local" means there is a limit on how fast information can be
transmitted, and that limit is the speed of light. Many Worlds is
completely consistent with that, in it there is no way you can send a
signal faster than light.*

 > *The other sectors are not just FAPP unoservable, they are not
observable in principle.*


*Hugh Everett didn't wake up one morning and say to himself, gee it would
be cool if there were a lot of different universes, I think I'll invent a
theory that has an infinite number of them. Instead he asked himself what
would happen if he took Schrodinger's Equation seriously and assumed it
really meant what it said, and when he did that those other universes just
popped up. The only way to get rid of them is to change Schrodinger's
equation as GRW has done, or to do what Copenhagen has done and say that
for some vaguely defined reason a vaguely defined thing called an
"observer" doesn't need to obey Schrodinger's Equation.  Many Worlds is
just bare bones, no nonsense Quantum Mechanics with no silly bells and
whistles pasted on. The only assumption Many Worlds makes is that the
mathematics means what it says.*

*Many Worlds logically explains a lot of stuff that seems bizarre to us,
yes it makes some predictions that can't be proven but it is hardly alone
in that. For example: we can't see things further away than 13.8 billion
light years because there hasn't been enough time for light to reach us,
and it's near as we can tell on the largest scale space is flat, if there
is any curvature at all it must be less than 0.4% so you'd need to go over
200 times 13.8 billion light years to form an unbounded sphere; so do you
really believe that there is nothing beyond 13.8 billion light years? If
you do then you must also believe the Earth really is the center of the
universe and 13.8 billion light years away there is a wall with absolutely
positively NOTHING on the other side. Do you believe that? If not why not?*

*> How could the presence of unobservable fairy tales affect anything at
> all?*



*But those "unobservable fairy tales" ARE observable, they are even
observable by Bruce Kellett, they are just not observable by you, with
"you"  defined as the person I'm talking to at this instant.  *

*John K ClarkSee what's on my new list at * *Extropolis*


bft

-- 
You received this message because you are subscribed to the Google Groups 
"Everything List" group.
To unsubscribe from this group and stop receiving emails from it, send an email 
to everything-list+unsubscr...@googlegroups.com.
To view this discussion on the web visit 
https://groups.google.com/d/msgid/everything-list/CAJPayv1qzoBEv2KHj_RYUzufbP29Kcy-t9KvVw_1cfXUOTgABQ%40mail.gmail.com.


Re: Is Many Worlds Falsifiable?

2023-08-31 Thread John Clark
On Thu, Aug 31, 2023 at 12:09 AM Bruce Kellett 
wrote:

*>> On Thu, Aug 31, 2023 at 12:27 PM smitra > > wrote:*
>
>
>>
>>
>>
>>
>>
>>
>>
>> *There is no problem here because in practice MWI is nothing more than
>> the usual QM formalism to compute the outcome of experiments where you then
>> assume that the ensemble of all possible outcomes really exists. Locality
>> then follows from the fact hat the dynamics of the theory is manifestly
>> local. The Hamiltonian only includes local interactions and observers are
>> part of this dynamics. Although observer are not explicitly treated as
>> being part of the wavefunction that describes the entire system, the
>> assumption is that in principle, this is the case. In practice, one can
>> then proceed according to the usual QM formalism.*
>>
>
> *> That is all very well, but it is not a local account of violations of
> the Bell inequalities.*
>

*Well of course it isn't! Bell's Inequality has been experimentally shown
to be violated, so if there are hidden variables they can't be local. *

> * Bell'e theorem applies in Everettian quantum mechanics in exactly the
> same way as it applies in one-world accounts.*
>

*Bell's Inequality applies to everything, even if Quantum Mechanics were
someday proved to be wrong Bell's theorem would still be valid. Bell didn't
need Quantum Mechanics to derive his inequality, he just needed logic and
high school algebra.  So even if  someday something supplanted Quantum
Mechanics that new theory would still have to conform to the fact
that Bell's Inequality is violated.*


> * >Bell's theorem proves that the effect is non-local, so no local account
> is possible in any interpretation of QM.*
>

*Exactly. And "local" means there is a limit on how fast information can be
transmitted, and that limit is the speed of light. Many Worlds is
completely consistent with that, in it there is no way you can send a
signal faster than light. *

 > *The other sectors are not just FAPP unoservable, they are not
observable in principle.*


*Hugh Everett didn't wake up one morning and say to himself, gee it would
be cool if there were a lot of different universes, I think I'll invent a
theory that has an infinite number of them. Instead he asked himself what
would happen if he took Schrodinger's Equation seriously and assumed it
really meant what it said, and when he did that those other universes just
popped up. The only way to get rid of them is to change Schrodinger's
equation as GRW has done, or to do what Copenhagen has done and say that
for some vaguely defined reason a vaguely defined thing called an
"observer" doesn't need to obey Schrodinger's Equation.  Many Worlds is
just bare bones, no nonsense Quantum Mechanics with no silly bells and
whistles pasted on. The only assumption Many Worlds makes is that the
mathematics means what it says.*

*Many Worlds logically explains a lot of stuff that seems bizarre to us,
yes it makes some predictions that can't be proven but it is hardly alone
in that. For example: we can't see things further away than 13.8 billion
light years because there hasn't been enough time for light to reach us,
and it's near as we can tell on the largest scale space is flat, if there
is any curvature at all it must be less than 0.4% so you'd need to go over
200 times 13.8 billion light years to form an unbounded sphere; so do you
really believe that there is nothing beyond 13.8 billion light years? If
you do then you must also believe the Earth really is the center of the
universe and 13.8 billion light years away there is a wall with absolutely
positively NOTHING on the other side. Do you believe that? If not why not?*

*> How could the presence of unobservable fairy tales affect anything at
> all?*



*But those "unobservable fairy tales" ARE observable, they are even
observable by Bruce Kellett, they are just not observable by you, with
"you"  defined as the person I'm talking to at this instant.  *

*John K ClarkSee what's on my new list at * *Extropolis*


bft




>
>

-- 
You received this message because you are subscribed to the Google Groups 
"Everything List" group.
To unsubscribe from this group and stop receiving emails from it, send an email 
to everything-list+unsubscr...@googlegroups.com.
To view this discussion on the web visit 
https://groups.google.com/d/msgid/everything-list/CAJPayv3OjRCtJM%3DroimoyRFJ4s0mpEpfjo8tiQsk6Aos4hHNpA%40mail.gmail.com.


Re: Is Many Worlds Falsifiable?

2023-08-30 Thread Bruce Kellett
On Thu, Aug 31, 2023 at 12:27 PM smitra  wrote:

> There is no problem here because in practice MWI is nothing more than
> the usual QM formalism to compute the outcome of experiments where you
> then assume that the ensemble of all possible outcomes really exists.
> Locality then follows from the fact hat the dynamics of the theory is
> manifestly local. The Hamiltonian only includes local interactions and
> observers are part of this dynamics. Although observer are not
> explicitly treated as being part of the wavefunction that describes the
> entire system, the assumption is that in principle, this is the case. In
> practice, one can then proceed according to the usual QM formalism.
>

That is all very well, but it is not a local account of violations of the
Bell inequalities. You merely claim that the local theory is such an
account, but you do not spell it out.

We have had this discussion before, and you couldn't give the detailed
local account then either. Bell'e theorem applies in Everettian quantum
mechanics in exactly the same way as it applies in one-world accounts.
Bell's theorem proves that the effect is non-local, so no local account is
possible in any interpretation of QM.

John points out the thought experiments by Deutsch makes it clear that
> the usual QM formalism will not work in certain cases, that will then
> falsify the ad hoc collapse postulate. If you then believe that MWI
> cannot account for violation of Bell's inequalities while ordinary QM
> can, then that begs the question of how removing the FAPP unobservable
> sectors where all other outcomes are realized, could matter at all.
>

The other sectors are not just FAPP unoservable, they are not observable in
principle. How could the presence of unobservable fairy tales affect
anything at all? The standard account of violations of the Bell
inequalities in quantum mechanics relies on the notion of non-locality. And
since the effect is non-local, no local account is possible.

Bruce

-- 
You received this message because you are subscribed to the Google Groups 
"Everything List" group.
To unsubscribe from this group and stop receiving emails from it, send an email 
to everything-list+unsubscr...@googlegroups.com.
To view this discussion on the web visit 
https://groups.google.com/d/msgid/everything-list/CAFxXSLTAg%3D%2BVf34FWWYXZ3%2BxKwkZmNWdt2VM_CRfHHBm3nqE%3Dg%40mail.gmail.com.


Re: Is Many Worlds Falsifiable?

2023-08-30 Thread smitra
There is no problem here because in practice MWI is nothing more than 
the usual QM formalism to compute the outcome of experiments where you 
then assume that the ensemble of all possible outcomes really exists. 
Locality then follows from the fact hat the dynamics of the theory is 
manifestly local. The Hamiltonian only includes local interactions and 
observers are part of this dynamics. Although observer are not 
explicitly treated as being part of the wavefunction that describes the 
entire system, the assumption is that in principle, this is the case. In 
practice, one can then proceed according to the usual QM formalism.


John points out the thought experiments by Deutsch makes it clear that 
the usual QM formalism will not work in certain cases, that will then 
falsify the ad hoc collapse postulate. If you then believe that MWI 
cannot account for violation of Bell's inequalities while ordinary QM 
can, then that begs the question of how removing the FAPP unobservable 
sectors where all other outcomes are realized, could matter at all.


Saibal





On 31-08-2023 00:55, Bruce Kellett wrote:

The many worlds idea has already been falsified because it cannot
account for the observed violation of the Bell inequalities for
entangled particles. MWI is supposedly a local theory -- where is the
local account of the correlations  of entangled particles?

Bruce

On Thu, Aug 31, 2023 at 12:39 AM John Clark 
wrote:


The short answer is yes, Many Worlds is falsifiable. For example,
right now there are experiments underway in an attempt to prove that
the GRW theory of objective quantum wave collapse makes predictions
that Many Worlds does not, if they are successful it will prove that
Everett was dead wrong, it's as simple as that.  GRW claims that
Schrodinger's equation is incomplete and that another very complex
term needs to be added to it because it's the only way they could
think of to get rid of all those worlds that for some reason they
dislike, there was simply no other reason to add that extra term.
With this new term Schrodinger's equation is no longer completely
deterministic because a random element is added such that the larger
the wave function is (the more particles it has) the more likely the
quantum wave function will objectively collapse. They carefully
tuned their very complex extra term inserted into Schrödinger's
equation in just such a way that, because an individual electron is
so small the probability of you being able to observe one
objectively collapse is almost but not quite zero; but the
probability of you NOT observing something as large as a baseball
NOT collapsing is also almost, but not quite, zero. Despite heroic
efforts. up to the present day nobody has found a speck of
experimental evidence in support of the GRW theory of objective
quantum wave collapse, and until and unless they do Many Worlds must
be the preferred theory according to Occam's razor because it makes
fewer assumptions, it has no need to complicate matters by adding
that extra term to Schrodinger's equation.

But GRW is not the only or even the most popular competitor to Many
Worlds, that honor would have to go to the Copenhagen
interpretation, and there is certainly no way to falsify that, but
back in 1986 in his book "The Ghost in the Atom" David Deutsch
proposed another way to falsify Everett's Many Worlds; the
experiment would be difficult to perform but Deutsch argues that is
not Many Worlds fault, the reason it's so difficult is that the
conventional view says conscious observers obey different laws of
physics, Many Worlds says they do not, so to test who's right we
need a mind that uses quantum properties.

In Deutsch's experiment, to prove or disprove the existence of many
worlds other than this one, a conscious quantum computer shoots
electrons at a metal plate that has 2 small slits in it. It does
this one at a time. The quantum computer has detectors near each
slit so it knows which slit the various electrons went through. The
quantum mind now signs a document for each and every electron saying
it has observed the electron and knows which slit it went through.
It is very important that the document does NOT say which slit the
electron went through, it only says that it went through one and
only one slit and the mind has knowledge of which one. Now just
before the electron hits the plate the mind uses quantum erasure to
completely destroy the memory of what slits the electrons went
through, but all other memories including all the documents remain
undamaged. After the document is signed the electron continues on
its way and hits the photographic plate. Then after thousands of
electrons have been observed and all which-way information has been
erased, develop the photographic plate and look at it. If you see
interference bands then the Many World interpretation is correct. If
you do not see interference bands then there are no worlds but this
one and the conventional interpretation is correct.

Deutsc

Re: Is Many Worlds Falsifiable?

2023-08-30 Thread Bruce Kellett
The many worlds idea has already been falsified because it cannot account
for the observed violation of the Bell inequalities for entangled
particles. MWI is supposedly a local theory -- where is the local account
of the correlations  of entangled particles?

Bruce

On Thu, Aug 31, 2023 at 12:39 AM John Clark  wrote:

> The short answer is yes, Many Worlds is falsifiable. For example, right
> now there are experiments underway in an attempt to prove that the GRW
> theory of objective quantum wave collapse makes predictions that Many
> Worlds does not, if they are successful it will prove that Everett was dead
> wrong, it's as simple as that.  GRW claims that Schrodinger's equation is
> incomplete and that another very complex term needs to be added to it
> because it's the only way they could think of to get rid of all those
> worlds that for some reason they dislike, there was simply no other reason
> to add that extra term. With this new term Schrodinger's equation is no
> longer completely deterministic because a random element is added such that
> the larger the wave function is (the more particles it has) the more likely
> the quantum wave function will objectively collapse. They carefully tuned
> their very complex extra term inserted into Schrödinger's equation in just
> such a way that, because an individual electron is so small the probability
> of you being able to observe one objectively collapse is almost but not
> quite zero; but the probability of you NOT observing something as large as
> a baseball NOT collapsing is also almost, but not quite, zero. Despite
> heroic efforts. up to the present day nobody has found a speck of
> experimental evidence in support of the GRW theory of objective quantum
> wave collapse, and until and unless they do Many Worlds must be the
> preferred theory according to Occam's razor because it makes fewer
> assumptions, it has no need to complicate matters by adding that extra term
> to Schrodinger's equation.
>
> But GRW is not the only or even the most popular competitor to Many
> Worlds, that honor would have to go to the Copenhagen interpretation, and
> there is certainly no way to falsify that, but back in 1986 in his book
> "The Ghost in the Atom" David Deutsch proposed another way to falsify
> Everett's Many Worlds; the experiment would be difficult to perform but
> Deutsch argues that is not Many Worlds fault, the reason it's so difficult
> is that the conventional view says conscious observers obey different laws
> of physics, Many Worlds says they do not, so to test who's right we need a
> mind that uses quantum properties.
>
> In Deutsch's experiment, to prove or disprove the existence of many worlds
> other than this one, a conscious quantum computer shoots electrons at a
> metal plate that has 2 small slits in it. It does this one at a time. The
> quantum computer has detectors near each slit so it knows which slit the
> various electrons went through. The quantum mind now signs a document for
> each and every electron saying it has observed the electron and knows which
> slit it went through. It is very important that the document does NOT say
> which slit the electron went through, it only says that it went through one
> and only one slit and the mind has knowledge of which one. Now just before
> the electron hits the plate the mind uses quantum erasure to completely
> destroy the memory of what slits the electrons went through, but all other
> memories including all the documents remain undamaged. After the document
> is signed the electron continues on its way and hits the photographic
> plate. Then after thousands of electrons have been observed and all
> which-way information has been erased, develop the photographic plate and
> look at it. If you see interference bands then the Many World
> interpretation is correct. If you do not see interference bands then there
> are no worlds but this one and the conventional interpretation is correct.
>
> Deutsch is saying that in the Copenhagen interpretation when the results
> of a measurement enters the consciousness of an observer the wave function
> collapses, in effect all the universes except one disappear without a trace
> so you get no interference. In the Many Worlds model all the other worlds
> will converge back into one universe when the electrons hit the
> photographic film because the two universes will no longer be different
> (even though they had different histories), but their influence will still
> be felt. In the merged universe you'll see indications that the electron
> went through slot X only and indications that it went through slot Y only,
> and that's what causes interference.
>
> I know that what I said in the above is a fair representation of what
> Deutsch was saying because some years ago I wrote to him about this and he
> said it was an accurate paraphrase.
>
> It must be admitted that like every theory Many Worlds makes predictions
> that cannot be tested, but a theory is n

Is Many Worlds Falsifiable?

2023-08-30 Thread John Clark
The short answer is yes, Many Worlds is falsifiable. For example, right now
there are experiments underway in an attempt to prove that the GRW theory
of objective quantum wave collapse makes predictions that Many Worlds does
not, if they are successful it will prove that Everett was dead wrong, it's
as simple as that.  GRW claims that Schrodinger's equation is incomplete
and that another very complex term needs to be added to it because it's the
only way they could think of to get rid of all those worlds that for some
reason they dislike, there was simply no other reason to add that extra
term. With this new term Schrodinger's equation is no longer completely
deterministic because a random element is added such that the larger the
wave function is (the more particles it has) the more likely the quantum
wave function will objectively collapse. They carefully tuned their very
complex extra term inserted into Schrödinger's equation in just such a way
that, because an individual electron is so small the probability of you
being able to observe one objectively collapse is almost but not quite
zero; but the probability of you NOT observing something as large as a
baseball NOT collapsing is also almost, but not quite, zero. Despite heroic
efforts. up to the present day nobody has found a speck of experimental
evidence in support of the GRW theory of objective quantum wave collapse,
and until and unless they do Many Worlds must be the preferred theory
according to Occam's razor because it makes fewer assumptions, it has no
need to complicate matters by adding that extra term to Schrodinger's
equation.

But GRW is not the only or even the most popular competitor to Many Worlds,
that honor would have to go to the Copenhagen interpretation, and there is
certainly no way to falsify that, but back in 1986 in his book "The Ghost
in the Atom" David Deutsch proposed another way to falsify Everett's Many
Worlds; the experiment would be difficult to perform but Deutsch argues
that is not Many Worlds fault, the reason it's so difficult is that the
conventional view says conscious observers obey different laws of physics,
Many Worlds says they do not, so to test who's right we need a mind that
uses quantum properties.

In Deutsch's experiment, to prove or disprove the existence of many worlds
other than this one, a conscious quantum computer shoots electrons at a
metal plate that has 2 small slits in it. It does this one at a time. The
quantum computer has detectors near each slit so it knows which slit the
various electrons went through. The quantum mind now signs a document for
each and every electron saying it has observed the electron and knows which
slit it went through. It is very important that the document does NOT say
which slit the electron went through, it only says that it went through one
and only one slit and the mind has knowledge of which one. Now just before
the electron hits the plate the mind uses quantum erasure to completely
destroy the memory of what slits the electrons went through, but all other
memories including all the documents remain undamaged. After the document
is signed the electron continues on its way and hits the photographic
plate. Then after thousands of electrons have been observed and all
which-way information has been erased, develop the photographic plate and
look at it. If you see interference bands then the Many World
interpretation is correct. If you do not see interference bands then there
are no worlds but this one and the conventional interpretation is correct.

Deutsch is saying that in the Copenhagen interpretation when the results of
a measurement enters the consciousness of an observer the wave function
collapses, in effect all the universes except one disappear without a trace
so you get no interference. In the Many Worlds model all the other worlds
will converge back into one universe when the electrons hit the
photographic film because the two universes will no longer be different
(even though they had different histories), but their influence will still
be felt. In the merged universe you'll see indications that the electron
went through slot X only and indications that it went through slot Y only,
and that's what causes interference.

I know that what I said in the above is a fair representation of what
Deutsch was saying because some years ago I wrote to him about this and he
said it was an accurate paraphrase.

It must be admitted that like every theory Many Worlds makes predictions
that cannot be tested, but a theory is not judged on the basis of what
predictions it makes that have neither been confirmed nor falsified
experimentally,  instead they are judged by how well they conform to
experiments that HAVE been performed, and in Many Worlds  case it conforms to
every physics experiment that has ever been made up to the present day. Yes
Everett's idea produces a lot of worlds, but Occam does NOT say the best
theory is the one that produces the simplest outcome, the be